Jump to content
 







Main menu
   


Navigation  



Main page
Contents
Current events
Random article
About Wikipedia
Contact us
Donate
 




Contribute  



Help
Learn to edit
Community portal
Recent changes
Upload file
 








Search  

































Create account

Log in
 









Create account
 Log in
 




Pages for logged out editors learn more  



Contributions
Talk
 



















Contents

   



(Top)
 


1 August 12  



1.1  Starting a political consultancy in India  
3 comments  




1.2  Purpose of cylinder at back of Renault FT-17 tank  
1 comment  




1.3  Jewish homosexuals adopting children  
6 comments  




1.4  Select Retail Holdings Limited  
4 comments  




1.5  Do non-Christians children have piggy banks?  
22 comments  




1.6  Marquis de Crecy 1800s  
3 comments  




1.7  collection of poems  
2 comments  




1.8  Split Duchies  
2 comments  




1.9  Value of a bee hive  
10 comments  




1.10  Working on my book again...  
22 comments  




1.11  How does SWIFT work?  
5 comments  




1.12  Oldest New Testament copy  
24 comments  






2 August 13  



2.1  Danish princes and princesses  
4 comments  




2.2  Meles Zenawi's skin so tan  
4 comments  




2.3  Satan, Lucifer and the Bible  
4 comments  




2.4  PRESIDENTIAL AND VICE PRESIDENTIAL SUCCESSION  
5 comments  




2.5  Less known cutting-edge authors from around the world  
8 comments  


2.5.1  North America  





2.5.2  South/Central America  





2.5.3  Great Britain  





2.5.4  Mainland Europe  





2.5.5  Africa  





2.5.6  Asia  





2.5.7  Australia  







2.6  Sri Lanka  
3 comments  




2.7  Blue Roofed Buildings  
5 comments  




2.8  Allied Bombing in WWII  
10 comments  




2.9  Polity IV and Freedom House  
1 comment  




2.10  Why does Paul Biya look so young  
11 comments  




2.11  More on US presedential succession  
9 comments  






3 August 14  



3.1  What sort of photographic processes are these?  
2 comments  




3.2  Best investment  
13 comments  




3.3  Cross-party voting in US federal elections  
12 comments  




3.4  Opposition to health care reform in the US  
27 comments  




3.5  olympic cities  
4 comments  




3.6  Robert Mugabe suspect of killing people?  
3 comments  






4 August 15  



4.1  Islamization in Ancient Mongolia  
4 comments  




4.2  Church use of popular music  
10 comments  




4.3  Abdoulaye Wade's famous shaven head  
3 comments  




4.4  What is the difference between empiricism and mechanistic materialism?  
4 comments  




4.5  Bangladesh student wing  
8 comments  




4.6  Christ killers  
13 comments  




4.7  Credit card merchant fees  
8 comments  




4.8  Dhabihah and hunting  
4 comments  






5 August 16  



5.1  What's REALLY the oldest profession?  
15 comments  




5.2  US Social Security numbers  
7 comments  




5.3  Are poor people in USA really buying homes?  
10 comments  




5.4  What cartoon is this describing?  
5 comments  




5.5  Hideous mermaid image  
2 comments  




5.6  Why'd they rename it Dominican Republic?  
9 comments  




5.7  What was Hitler's opinion of Republicanism?  
16 comments  




5.8  List of Libraries  
6 comments  




5.9  Using debit or credit card in supermarket - how much info can they get about me?  
2 comments  




5.10  Agricultural Subsidies  
4 comments  




5.11  UK's action in Turks and Caicos Islands  
3 comments  




5.12  Is there really a TV news station?  
2 comments  






6 August 17  



6.1  List of Arts and Sciences  
1 comment  




6.2  Indonesian ballot paper of 1955  
1 comment  




6.3  meaning of ww2 japanese submarine designations  
2 comments  




6.4  Looking for a Captain Cook work  
3 comments  




6.5  Tracing the movements of George VI of the United Kingdom  
5 comments  




6.6  Terms of the lease of the New Territories, Hong Kong  
4 comments  




6.7  White vs. APL family  
6 comments  




6.8  islamic monks?  
3 comments  




6.9  Elections of Kenya and mozambique  
3 comments  




6.10  Does Congress (in the US) have a source control system for the actual words going into a bill?  
16 comments  




6.11  Mexican vs. white interracial marriage  
8 comments  




6.12  citing music  
1 comment  




6.13  Judaism Ethnic divisions vs. denomination  
2 comments  




6.14  Districts of West Bengal versus religion  
2 comments  




6.15  North Americal Aboriginal Treaties  
5 comments  




6.16  Caucasian Eyes  
2 comments  




6.17  rotten vegetables at protests and theater  
2 comments  




6.18  Danish titles  
3 comments  




6.19  Logical fallacy: Good or bad based on who does it?  
7 comments  




6.20  Is there any religion where there are two good Gods?  
3 comments  




6.21  Ethnic groups of India vs. Buddhism  
1 comment  




6.22  Bangladeshi Buddhist  
1 comment  




6.23  Wikipedia sentient?  
2 comments  















Wikipedia:Reference desk/Humanities







 

Edit links
 









Project page
Talk
 

















Read
Edit
Add topic
View history
 








Tools
   


Actions  



Read
Edit
Add topic
View history
 




General  



What links here
Related changes
Upload file
Special pages
Permanent link
Page information
Get shortened URL
Download QR code
Wikidata item
 




Print/export  



Download as PDF
Printable version
 




Print/export  



















Appearance
   

 






From Wikipedia, the free encyclopedia
 

< Wikipedia:Reference desk

This is an old revision of this page, as edited by 209.244.187.155 (talk)at17:30, 18 August 2009 (Wikipedia sentient?). The present address (URL) is a permanent link to this revision, which may differ significantly from the current revision.
(diff)  Previous revision | Latest revision (diff) | Newer revision  (diff)

Welcome to the humanities section
of the Wikipedia reference desk.
Select a section:
Want a faster answer?

Main page: Help searching Wikipedia

   

How can I get my question answered?



How do I answer a question?

Main page: Wikipedia:Reference desk/Guidelines

See also:



August 12

Starting a political consultancy in India

Hello. I wish to know what is the status of political consultancy in India. Are there established players in the field which provide assistance to poltical parties on policies, strategy, voter issues, demographics etc.? I actually want to start such a consultancy in India (in a few years time) and wish to know what are the main issues I should take care of / focus on. Basically how to go about doing it. I know this is a pretty big question to ask on the refdesk but I hold you people in high esteem, and I'm sure you'll be able to help. Aside: I actually came up with the idea of a political consultancy before I knew such organizations already existed. My sad life is full of entrepreneural ideas that someone else had already thought of :( My motivation in starting a political consultancy in India is altruistic first and profit-seeking second - I hope to make political parties more centered on real issues that affect voters. Thanks. --ReluctantPhilosopher (talk) 07:42, 12 August 2009 (UTC)[reply]

You are unlikely to find someone on the Reference Desk with expertise in this field. The experts are busy working for politicians. My recommendation would be the following: Before you start your own consultancy, spend a few years working for an established consultant whose work you respect. I would think that in this field, experience and contacts are key. The experience would be the best way for you to be able to answer the questions you have posed, which really no one else could answer for you. To find work with an established consultant, you will first have to identify them. Sometimes they are mentioned in news articles. Besides this, you might try contacting the offices of politicians whom you admire and ask if you might have an "informational interview" on careers in the field with a member of the politician's staff. Your objective at these informational interviews should be to learn more about the field, but, maybe more important, to put together lists of names (for example, of consultants or their staff) to contact for further information gathering. Eventually, you are likely to find someone who knows of a position (and might even recommend you for it) or possibly someone who wants to hire you. Otherwise, you can approach consultants with whom you'd like to work to try to convince them to hire you. Marco polo (talk) 12:41, 12 August 2009 (UTC)[reply]

I am interested in American lobbying. My college major was politcal science and then I attended law school. During the academic year and summers, I interned as a law clerk with a Senate subcommittee. It also helped to be active in political campaigns for office and to advocate for issues that were personally important to me. Most people interested in politics concentrate their studies in political science or history. I found that people involved in the field are happy to share information. Communications is another effective major. Personal networking, as Marco Polo recommended, is crucial. 75Janice (talk) 02:07, 13 August 2009 (UTC)75Janice[reply]

Purpose of cylinder at back of Renault FT-17 tank

What is the purpose of the "tail" apparatus on this Renault FT-17 tank, including the cylinder and chains? (This tank is exhibited in the Compiègne Forest at the location where the armistice of the First World War was signed.)

From the side
From the back

David.Monniaux (talk) 10:27, 12 August 2009 (UTC)[reply]

Question moved to the Science RD: http://en.wikipedia.org/wiki/Wikipedia:Reference_desk/Science#Purpose_of_cylinder_at_back_of_Renault_FT-17_tank

Jewish homosexuals adopting children

Is it possible to Jewish homosexual couples to adopt a child? Will this child be Jewish? Since Jewish implies having a Jewish mother, I suppose that male homosexuals will have a problem...And what if only one woman in a female homosexual couple is Jewish? Since we don't know which one is the mother and which one is the father, it could be argued that the non-Jewish is the mother, and therefore, the child is not Jewish.--Quest09 (talk) 10:37, 12 August 2009 (UTC)[reply]

Yes but if the Jewish one turns out to be a real mother you will be wrong then probably. Also if they both want to be the father it will complimacate things. ~ R.T.G 10:57, 12 August 2009 (UTC)[reply]
If you look at our article Who is a Jew?, you will see that a person does not need to be born to a Jewish mother to become a Jew. There is also the possibility of conversion. I think that if a homosexual couple wanted to raise a child as a Jew, there would not be a problem, assuming that at least one of them belongs to an accepting Jewish congregation. The Jewish (adoptive) parent would be welcome to bring his or her adoptive child to the synagogue, where the child could undertake the study needed for conversion to that denomination of Judaism. Upon conversion, the child becomes Jewish. Marco polo (talk) 12:30, 12 August 2009 (UTC)[reply]

I keep hoping Sammy Davis, Jr. will come back to life and set the record straight: Judaism is a religion. Believe it, and you, too, can become a Jew. Don't believe it, and it doesn't matter one bit who's your mama or who's your daddy. DOR (HK) (talk) 02:45, 13 August 2009 (UTC)[reply]

As far as being "born Jewish", wouldn't the religion of the actual mother figure into it? And if the mothere was not Jewish, then presumably the child could convert at some point. Baseball Bugs What's up, Doc? carrots 08:35, 13 August 2009 (UTC)[reply]
The Nazis seemed to have a rather different view, DOR (HK). They got rid of many people who were Christians, atheists, Lutherans or what have you, purely because they had one or more Jewish ancestors back to the 4th generation, and they were thus considered to still be too Jewish to be tolerated. Their current religion didn't come into it. But you're right, one can convert to Judaism from any other faith. Which means Jewishness is not just about genetics, but equally it's not just about religion. -- JackofOz (talk) 13:08, 13 August 2009 (UTC)[reply]

Select Retail Holdings Limited

Hi, this company has taken over one of the largest supermarkets in Ireland. I can't find anything to base an article on, even a stub. No website no real facts except that they own this supermarket chain (Superquinn). Are they English or Irish etc? Do they own other companies? What made them succesful enough to take on an important supermarket chain? We could have at least a stubby article on this one so if anyone knows a good source of info please share. ~ R.T.G 10:54, 12 August 2009 (UTC)[reply]

This article from the Indo at least names the members of the consortium. Seem to be a group of Irish investors. Fribbler (talk) 11:09, 12 August 2009 (UTC)[reply]
They're registered with the Companies Registration Office, which gives their address as SCD House, Waterloo Road, Dublin 4. It says the company was registered on 07/11/2002 under another name, and changed name in 2005. You can download more detailed info from http://www.cro.ie, but they want a (fairly nominal) sum for each document. From the free info, you really can't tell very much, and that 2002 registration doesn't mean the company as it exists now has been trading since then - accountants often register companies and then sell them on as off-the-shelf companies (it's a perfectly legitimate thing to do), so you can't tell when the current owners acquired it unless you download the company info sheets. There is no registration under that name in the UK's Companies House. It's likely that the company is nothing more than a holding company, as its name suggests - that it isn't an existing retailer, but is instead just a group of investors and managers formed for the purpose of owning and running that supermarket. -- Finlay McWalterTalk 11:11, 12 August 2009 (UTC)[reply]
That will make a start lads thanks. If anyone comes across more please add. ~ R.T.G 11:20, 12 August 2009 (UTC)[reply]

Do non-Christians children have piggy banks?

I mean, pigs have a symbolic meaning utterly different in different parts.--Quest09 (talk) 11:16, 12 August 2009 (UTC)[reply]

Onpiggy bank it says that the piggy bank has both English and Indonesian appearances independantly. ~ R.T.G 11:19, 12 August 2009 (UTC)[reply]
So, vaguely related, I wonder if other countries use the synonym for wages, "bringing home the bacon"? Baseball Bugs What's up, Doc? carrots 13:19, 12 August 2009 (UTC)[reply]
I can't imagine that there are any restrictions on the children of atheist parents using a piggy bank. Googlemeister (talk) 14:19, 12 August 2009 (UTC)[reply]
I certainly used to have a piggy bank, and have never been a Christian. Algebraist 14:54, 12 August 2009 (UTC)[reply]
Your answer is the closest thing to a real answer to his question. If one exception is sufficient, then the answer to his question is, "Yes." Baseball Bugs What's up, Doc? carrots 19:11, 12 August 2009 (UTC)[reply]
What do piggy banks have to do with Christianity? Just because a Jew or a Muslim won't eat pork doesn't mean they can't keep their change in an attractive, pig-shaped bank. They don't think pigs are evil or unlucky or anything. --98.217.14.211 (talk) 14:59, 12 August 2009 (UTC)[reply]
Hard to tell what exactly the OP is getting at, but I read it as referring to the pig as a symbol of greed, and that by "Christian" he may have meant "Western". Tempshill (talk) 16:21, 12 August 2009 (UTC)[reply]
I would think it was more along the line that Muslims consider them unclean, whereas Westerners consider them delicious. TastyCakes (talk) 16:32, 12 August 2009 (UTC)[reply]
And presumably westerner muslims live in constant cognitive dissonance. Algebraist 16:33, 12 August 2009 (UTC)[reply]
Not to mention vegetarians of all stripes ;) TastyCakes (talk) 16:42, 12 August 2009 (UTC)[reply]
There are Christians/Westerners who don't eat pork (SDA). 92.82.114.80 (talk) 16:50, 12 August 2009 (UTC)[reply]
Anyway, pigs are amusing, even if you don't eat them. I think that's the basic issue in regards to the banks. --98.217.14.211 (talk) 17:04, 12 August 2009 (UTC)[reply]
Pigs are a symbol of fertility and often the staple livestock in East Asia - so yes, lots of piggy banks there, often garishly painted gold --PalaceGuard008 (Talk) 02:44, 13 August 2009 (UTC)[reply]

The OP seems to be starting with the assumption that all non-Christians are either Jewish or Muslim. Perhaps that’s the case where he/she lives, but it must be a pretty odd place.DOR (HK) (talk) 02:53, 13 August 2009 (UTC)[reply]

All is an exaggeration, but the "vast majority" of British Asians in Bradford have Pakistan or Kashmir as their origin, and some of the remainder will be Bangladeshi in origin. Admittedly the Bradford article does note sizeable Hindu and Sikh populations too. AlmostReadytoFly (talk) 08:27, 13 August 2009 (UTC)[reply]
How did you know the OP was from Bradford? --PalaceGuard008 (Talk) 07:30, 14 August 2009 (UTC)[reply]
I was wondering the same thing. My assumption is that he is an admin and has access to the OP's IP adress. TomorrowTime (talk) 08:25, 14 August 2009 (UTC)[reply]
Admins don't have access to users' IP addresses - only WP:CheckUsers can see them, and they can only look at them under particular circumstances - not just to satisfy curiousity. Warofdreams talk 10:26, 14 August 2009 (UTC)[reply]
That puts me at ease a bit. I'll admit I don't particularly like the idea of admins having free access to IPs. TomorrowTime (talk) 11:02, 14 August 2009 (UTC)[reply]
I'm not an admin or a checkuser, I was merely pointing out that that idea of a place where the vast majority of non-Christians are Muslims is not so odd. AlmostReadytoFly (talk) 11:04, 14 August 2009 (UTC)[reply]

Here is a "kosher piggy bank" for sale: [1] -- Mwalcoff (talk) 22:27, 16 August 2009 (UTC)[reply]

Marquis de Crecy 1800s

An ancestor, James Binion-Cooper (1789-1851) was an ironfounder in London. According to family legend he was in business with the Marquis de Crecy. I am seeking information. I have found Alice de Crecy, widow of French Baron in 1881 London census. Had her husband died in France? Was he a son of Louis -Alexandre Verjus of France. Had he gone to London after the French revolution?202.154.150.113 (talk) 11:30, 12 August 2009 (UTC)[reply]

See Louis de Verjus. His son, Louis-Alexandre Verjus, marquis de Crécy, was born in 1676 and died in 1763, so he would not have been alive after the French Revolution. There were other Marquises (I guess that's the plural) who succeeded him, so you would need to find out which one was alive during the Revolution. Who then was a gentleman? (talk) 19:43, 12 August 2009 (UTC)[reply]
The plural of a French word ending in 's' or 'x' is the same: un marquis, deux marquis. —Tamfang (talk) 23:16, 17 August 2009 (UTC)[reply]

collection of poems

i am looking for a large collection of poems which can be sorted by theme, time period, etc. —Preceding unsigned comment added by 218.186.12.233 (talk) 15:12, 12 August 2009 (UTC)[reply]

Oxford Book of English Verse? --TammyMoet (talk) 16:55, 12 August 2009 (UTC)[reply]

Split Duchies

Does any body know the word for a duchy that is split among more than one son after a dukes death? Like the Ernestine duchies, the Schleswig-Holstein-Sonderborg duchies, and the Bavarian duchies. I thought it was stem duchy until I read the article.

Technically appanages if the heir ruled before the death of the duke. After the death of the duke, only the younger dukes held appanages and the heir held the remainder as duke. I can't find an English jargon word for the split duchies so you could use any descriptive word such as partition or division.
Sleigh (talk) 17:17, 15 August 2009 (UTC)[reply]
Do you mean a retrospective word for the previous duchy, before the split? —Tamfang (talk) 23:17, 17 August 2009 (UTC)[reply]

Value of a bee hive

My Grandfather recently discovered he has a bunch of bees living in a hollow tree on his property. Is there any value to these (IE. Can he sell the rights to someone who will then come and collect them)? He lives right around the KY-OH-IN border area. Thanx, 71.67.139.110 (talk) 17:27, 12 August 2009 (UTC)[reply]

No. They can only be easily removed when they are swarming, and even then I don't think it's normal for any money to change hands.--Shantavira|feed me 17:33, 12 August 2009 (UTC)[reply]
Is there any use for them at all? 71.67.139.110 (talk) 17:35, 12 August 2009 (UTC)[reply]
Just like bees in an artificial hive, they will pollinate surrounding plants. While true that you won't get any honey or wax from them, most of the money made by modern commercial beekeepers is from renting the hives out to farmers as pollinators, and the honey/wax/royal jelly/etc. is just extra on top of that. There is actually a large number of different bee breeds, so the otherwise-unspecified wild bees ("mutts") in your tree aren't of particular value - especially if there is concern that they might be africanized. Most beekeepers probably won't pay you to take your bees away - if anything they might charge you to do it, though many will probably do it for free, as a public service. -- 128.104.112.100 (talk) 17:46, 12 August 2009 (UTC)[reply]
Maybee he could just let them bee? Tee hee. --98.217.14.211 (talk) 18:10, 12 August 2009 (UTC)[reply]
Do-Bee or Don't-Bee, that is the question. Baseball Bugs What's up, Doc? carrots 19:09, 12 August 2009 (UTC)[reply]
It's not relevant to the OP but modern beekeepers in the US charge farmers like that, British beekeepers usually keep them in one place and just sell the honey. --Tango (talk) 18:18, 12 August 2009 (UTC)[reply]
Tango, do you have a reference for this surprising fact? Or would looking one up slow you down in your crusade to combat the insidious American Bias? A simple internet search for "pollination services" shows a good number of beekeepers in the UK that offer these kinds of services.
At first I thought that you were thinking that Britain does not have the vast industrial farms of the American midwest, but I'm sure you understand that not all of the USA is like that, and that there are areas OUTSIDE the USA that are. APL (talk) 00:40, 13 August 2009 (UTC)[reply]
I can go and find a reference if you like, but my actual source is personal knowledge from discussions with my Grandfather who has been keeping bees for decades. If you read my comment carefully you will see I said "usually", I'm sure there are exceptions. --Tango (talk) 18:51, 13 August 2009 (UTC)[reply]
My Grandfather is a beekeeper and he usually collects swarms for free in exchange for keeping the bees. I'm not sure about nests, but I can't see him paying anyone for them, at best he would do it for free and keep the bees. They won't be worth much because you can't offer any assurances that they are good quality bees. --Tango (talk) 18:18, 12 August 2009 (UTC)[reply]

Working on my book again...

A while back, I asked a question for a christian fantasy book I was writing. I've hit another speedbump, and I need some help. Does anyone know of a species of demon (not one specific demon) that could be considered the high guards of hell? Any answers will be much appreciated (One other thing: will I get sued if I make hell ruled by a triumvirate?) Library Seraph (talk) 18:42, 12 August 2009 (UTC)[reply]

Sued by who? One of Satan's lawyers? (Pardon the possible redundancy) Baseball Bugs What's up, Doc? carrots 19:07, 12 August 2009 (UTC)[reply]
Ho ho, you made me chuckle nicely. Cheers. Prokhorovka (talk) 19:11, 12 August 2009 (UTC)[reply]
There's an outside chance you could get sued by D.C. Comics or (less likely) Neil Gaiman, who depicted a (temporarily) triumvirate-ruled hell in his Sandman series, but only if your version resembled his in detail; the mere concept is too general to fall under copyright (literary ideas are not in themselves copyrightable), and anyway may well not be original to Gaiman. [Update: it seems to have originated in earlier D.C. Comics works which Gaiman was utilising as back-story, but that's not to say that those stories originated the concept, either.)
Aside from the low probability of actual legal action, there is also the possibility that some readers may assume you lifted the idea from Gaiman [in particular or the D.C. Universe more generally], and adversely criticise you, with deleterious effects to your reputation and sales. However, all professional writers and editors know that broad ideas become common property and that writers are inspired by and riff off each others' ideas all the time, it's how well you make fresh use of them that counts.
At this point you may now wish to scrupulously avoid learning anything more about Gaiman's version, to ward off any possibility of unconscious influence or unjust suggestion of plagiarism. Alternatively, you might go ahead and read the Sandman series in order either to avoid any further accidental similarities at all, or to make explicit allusion to it, thus paying hommage. Again, writers frequently do this: for example, I've just finished reading a novel by Arthur C Clarke & Stephen Baxter in which advanced quantum wormhole technology makes it easy to see anything, anywhere (I'll avoid further spoilers) and examines the far-reaching consequences for human society. This was ultimately inspired by the late Bob Shaw's stories published in the 1960s and '70s in which he used the concept of "slow glass" to the same end. Clarke and Baxter avoid any possible criticisms from those who don't understand the dynamics of literature by dedicating their book to Shaw, using the title of his first slow-glass story - [The] Light of Other Days - as that of their novel, and mentioning it and him in their Afterword. 87.81.230.195 (talk) 20:27, 12 August 2009 (UTC)[reply]
You might check around the article Christian demonology. It is quite complex. Classification of demons may be of some help. I don't see a lot of classification by species—they all seem to be just "demons" though they are individually of different rank and specialization. --98.217.14.211 (talk) 19:24, 12 August 2009 (UTC)[reply]
Dante's Inferno would have a derth of information for you. Livewireo (talk) 20:28, 12 August 2009 (UTC)[reply]
I think you mean plethora. Who then was a gentleman? (talk) 20:31, 12 August 2009 (UTC)[reply]
...yes. Derth is the opposite of what I mean. This is what I get for sleeping in class. Livewireo (talk) 20:37, 12 August 2009 (UTC)[reply]
I hope it wasn't a spelling class.  :) -- JackofOz (talk) 21:06, 12 August 2009 (UTC)[reply]
Maybe the class was in the 16th century, when "derth" was still an acceptable variant? Algebraist 21:18, 12 August 2009 (UTC)[reply]
...though not of plethora.--Wetman (talk) 22:12, 12 August 2009 (UTC)[reply]
Bulbous bouffant! —Tamfang (talk) 23:19, 17 August 2009 (UTC)[reply]
Of course the Monster Manual would be of use here. The First Edition actually sorts the demons and devils by level. They used a variety of source material and, of course, made up some stuff. Tempshill (talk) 23:32, 12 August 2009 (UTC)[reply]
There is a role-playing game In Nomine full of stuff like this. You can probably find the demon species with a web search. 70.90.174.101 (talk) 01:50, 13 August 2009 (UTC)[reply]

Rather than a triumvirate, why not a five-person body?DOR (HK) (talk) 03:03, 13 August 2009 (UTC)[reply]

Lawyers. --Dweller (talk) 11:31, 13 August 2009 (UTC)[reply]

I might make my hell ruled by a different number of demons than just 3 (Although not to do with the sigil of Baphomet, because anythinbg to do with satanism makes me feel like throwing up). As for the high guard, I guess I'll have to go look myself. I was kind of hoping that I wouldn't have to go digging through the demon articles again *Sigh* Library Seraph (talk) 14:05, 13 August 2009 (UTC)[reply]

You're writing about demons in hell but satanism is a problem? Adam Bishop (talk) 14:29, 13 August 2009 (UTC)[reply]

Since you're writing a Christian fantasy, it may not be much help; but guards of hell apparently exist in Islamic tradition, though they're angels (assigned to prevent escape?) rather than demons. According to Hell#Islam, 'The gate of Hell is guarded by Maalik who is the leader of the angels assigned as the guards of hell also known as Zabaaniyah." Deor (talk) 14:42, 13 August 2009 (UTC)[reply]

Thanks Deor, I may look into that; I have been drawing inspiration from other mythologies. Oh, and Adam, I'm writing a christian fantasy; the reason I haven't asked any questions abotu the angels (who are my priotagonists) is because I'm finding them easier to write about than the demons. Library Seraph (talk) 15:00, 13 August 2009 (UTC)[reply]

Library Seraph, to relieve your nausea, bear in mind that most supposed "satanism" was/is no less literary invention (sometimes by people who believed their own fantasies) than your own work. Very few people actually practiced "satanism" in the sense you use the term, and many of those who did were just the punks of their era, out to shock. Consider, too, that the concept of "Satan" as an evil entity was itself a largely literary-theological invention, as modern Christian theologians such as Elaine Pagels (see her The Origin of Satan) acknowledge. The original "satan" was merely a loyal servant of the Hebrew God who carried out the latter's tasks on Earth, which sometimes involved testing humans with adverse circumstances. This was conflated with poetical references to the (actual or hoped-for) downfall of a human ruler (Nebuchadnezzar II?) - "How art thou fallen, oh Lucifer, son of the morning" as I recall, where "Lucifer" was in literal terms the morning apparitions of the planet Venus - to form the basis of a whole fantasy history about rebellion in heaven, elaborate demonic hierarchies in Hell, and so on. 87.81.230.195 (talk) 01:12, 14 August 2009 (UTC)[reply]

Now that i think about it, I may not actually have a triumvirate. I think that term usually implies a bit more co-operation, and less outright conflict that what I have. Sorry for any trouble I put anyone through Library Seraph (talk) 00:00, 16 August 2009 (UTC)[reply]

How does SWIFT work?

I am wondering how SWIFT works?

I mean say for example a Bank in Russia (which has no presence in Canada) is getting funds from a Bank in Canada (which has no presence in Russia).

Also note that no Paper money changes hands. (Banks dont do most of their business now a days with Paper money contrary to the popular belief)

So why would a Russian bank accept a wire transfer from the Canadian bank?

In other words, the Russian bank will be making money for the new client based on the fact that this new client owns money abroad.

Are there some sort of regulations like say from the Reserve Bank of each country to facilitate this kind of transaction?

The same question goes for Western Union which uses the CHIPS system. How does it actually work?

Anyways I am also looking to read some books on the banking systems and how they work. Recommendations welcomed.

It seems to me that the banking industry is a secret. No university teaches courses on how they really work. Like Bank Drafts, Certified Cheques, Money Orders, Wiring funds etc. They do teach Finance, Economics, and Business, but not Banking. I think all the people who work for banks learn about it as they work, not in books.

--33rogers (talk) 18:59, 12 August 2009 (UTC)[reply]

Member institutions own CHIPS, and it maintains a very simple account (I think on a daily basis). Membership is contingent on a bank having met strict checks. When bank A sends to bank B, CHIPS merely keeps a running total. In practice, for large banks, the amount of transactions in one direction roughly balances - if it does not, after a given accounting period, the bank that is in debt to the system must compensate it, often with a physical instrument or the (notional) transfer of bullion (a great deal of the gold held in the United States Bullion Depository is owned by banks; to settle the debt some portion need only be reassigned from one bank to the other - nothing need be physically moved). Smaller banks aren't members of these schemes, and so instead maintain accounts at larger banks that are. For example, I've CHIPSed money from an account in a credit union in California to a bank in Scotland, and the paperwork shows Natwest as the intermediary. -- Finlay McWalterTalk 19:47, 12 August 2009 (UTC)[reply]
University-level subjects on banking are usually taught under Finance or Law, depending on whether you are more interested in the monetary or the legal aspects of it. Googling turns up a number of subjects which may interest you. --PalaceGuard008 (Talk) 02:41, 13 August 2009 (UTC)[reply]

On the secrecy angle, would you also consider insurance, real estate, consulting, truck driving and circus management to be “secret”? None of them are taught as subjects per se. DOR (HK) (talk) 03:07, 13 August 2009 (UTC)[reply]

SWIFT and CHIPS are fundamentally different systems. SWIFT provides a secure and reliable network for the exchange of financial instructions and other messages between its members, but SWIFT member firms do not hold money in accounts at SWIFT. If a Canadian bank is paying a Russian bank then the two banks must either have nostro accounts with each other (i.e. be correspondent banks) or must each hold accounts at a third-party agent bank. CHIPS is a netting engine, so CHIPS runs intraday balances for its members which are then settled at the end of the day through accounts at the Federal Reserve Bank. So SWIFT can carry payment instructions in any currency, whereas CHIPS can only process payments in US dollars. All of this information is available at www.swift.com and at www.ffiec.gov. Gandalf61 (talk) 13:09, 13 August 2009 (UTC)[reply]

Oldest New Testament copy

Where is the oldest New Testament, especially books of Matthew, Mark, Luke, John or Acts. What year are they? How do they know the age (DNA, carbon testing, or what)? Have qualified scientist verified the age? Where are they where one could view them? --LordGorval (talk) 23:23, 12 August 2009 (UTC)[reply]

Useful snippet of an article... there are lots of books in the New Testament, and different books have different ages (and different fragments that survive). Nothing is older than a century after it was written (only copies of copies of copies survive, nothing original). The oldest piece is a snippet of the gospel of John (Rylands Library Papyrus P52) that dates to around 125. There are a number of different ways that it is dated, including scientific and textual. --98.217.14.211 (talk) 23:45, 12 August 2009 (UTC)[reply]
The oldest substantially complete manuscripts (as opposed to fragments) are from the 4th century, such as Codex Sinaiticus... AnonMoos (talk) 00:47, 13 August 2009 (UTC)[reply]
One important method of estimating the age of a manuscript is to use Palaeography, the study of ancient writing. In most languages, different letters, different ways of writing individual letters, and different overall styles of handwriting go in and out of fashion, so by comparing many examples of ancient texts it's possible to work out roughly what was being used when (much as with the changing styles and techniques of ancient pottery). Similarly, actual vocabulary changes as old words fall into disuse and new ones are coined or introduced from elsewhere, and studies can establish roughly (or occasionally more exactly) when given changes occurred. The ideas expressed in texts are also subject to change and development, and their evolution through time can often be estimated.
A new textual example or fragment can therefore be compared with the existing body of knowledge to suggest approximately when it was written. Obviously this can rarely give an exact age, as some scribes would have been more conservative than others, and with no universal fast communications some innovations or ideas would spread unevenly.
These methods are of course dependent on degrees of assumption, are limited by what texts are available and have been studied (so new ones can change our understanding of how early or late some factor appeared or died out), and cannot themselves eliminate the possibility of faking by someone as knowlegeable as the examining scholars. 87.81.230.195 (talk) 03:16, 13 August 2009 (UTC)[reply]
The "original version" of the Bible is now online (this is the Codex Sinaicus referred to above). Read more about it in this BBC article. Jørgen (talk) 07:05, 13 August 2009 (UTC)[reply]

Where is the the oldest complete Latin version and the oldest complete English version? --LordGorval (talk) 11:52, 13 August 2009 (UTC)[reply]

Codex Amiatinus is the oldest Latin one. Adam Bishop (talk) 14:23, 13 August 2009 (UTC)[reply]
Parts of Biblical books were translated into Old English (some excerpts allegedly by King Alfred himself), but the earliest complete translation into English was by Wycliffe and his followers in the 14th century... AnonMoos (talk) 15:09, 13 August 2009 (UTC)[reply]
Which English? Old English? Middle English? I am not sure if people who only know modern English would be able to read those languages. For example, here is a passage from the Middle English version called the Ormulum.
Forrþrihht anan se time comm
þatt ure Drihhtin wollde
ben borenn i þiss middellærd
forr all mannkinne nede
he chæs himm sone kinnessmenn
all swillke summ he wollde
& whær he wollde borenn ben
he chæs all att hiss wille.
I can't read it, but it is Middle English. It is not complete and is from the 12th century. As noted above the Wycliff version is the earliest known complete version. Googlemeister (talk) 15:18, 13 August 2009 (UTC)[reply]
Whatever -- if you're not willing to consider 14th-century English to be "real" English, then the first translation into "real" English (according to your personal definition) is logically the first translation which was made after the date you have chosen to assign to the beginning of your "real" English... AnonMoos (talk) 16:07, 13 August 2009 (UTC)[reply]
In my opinion, it isn't "I can't read it". It is "I refuse to see if I can read it." Simply reading it phonetically, it looks like "For bright in the time come that our Drighton would ben born in this middle lord for all mankind need. He chose him some kinsmen all we like some he would and where he would born been he chose all at his will." That is not perfect modern English, but it is rather sensible. In fact, it is very close to the accepted translation. -- kainaw 20:22, 13 August 2009 (UTC)[reply]
I would have thought that Tolkien had penetrated far enough into the public consciousness that『middellærd』would ring a bell. Deor (talk) 22:33, 13 August 2009 (UTC)[reply]
My point is valid. We are not talking about something that is comprehensible but contains odd words and phrasings such as you would have from say the period of Shakespeare, this stuff looks more German then English. If you have an archaic form of a language that is incomprehensible to modern speakers of the language, you are going to have a hard time convincing people that it is in fact the same language.
Addendum, the sample I provided was 12th century, not 14th. Googlemeister (talk) 16:16, 13 August 2009 (UTC)[reply]
It is easy to see that the language that extract is written in is not mutually intelligible with Modern English, so it is a different language. The problem comes in working out where to draw the line between Middle English and Modern English. There is a standard date for that, I believe, but it is pretty arbitrary. The language changed gradually so Late Middle English and Early Modern English would have been mutually intelligible. --Tango (talk) 17:35, 13 August 2009 (UTC)[reply]
There are so many different phases of Middle English it's not really fair to compare the Ormulum with Wycliffe. Wycliffe is certainly hard to read, but it is fairly intelligible, like Chaucer, though not as recognizably English as Shakespeare. The Ormulum is only Middle English in the sense that it came after the Conquest. It was barely influenced by French, if at all, and at the time and place where it was written it is really pretty much Old English. (Though it does have lots of Norse in it, which would also be an important part of Middle English.) In any case, we can all agree that Wycliffe is English and his Bible is the first complete translation, right? Adam Bishop (talk) 20:09, 13 August 2009 (UTC)[reply]
Since Ormulum is not complete, yes I would agree the Wycliffe is the oldest complete English translation of the New Testament that is known regardless of whether the language used in the Ormulum is English. Googlemeister (talk) 20:39, 13 August 2009 (UTC)[reply]
It should be noted that using 'hard' science (such as carbon dating) no document in existance has been dated to before 1200AD. On most things like this carbon dating is accurate to about +/- 700+ years, the error rating given is the error of the lab extracting the C14, not the error in what that C14 actually means for the dating. This is something that few people understand. For example a sample might come back and say 1300BC +/- 200 years. Thats 200 years error at the lab who is trying to measure the amount of C14, it says nothing about the error in calculating what amount C14 should have been present at all the years intermediate years. And it says nothing about when the wood used actually used to write on, just when the tree died, and it says nothing about how carbon could have been absorbed by the paper over 2,000 years etc. This is why carbon dating has so little weight in historical circles. Its useless. I would take anything before 1500AD with a grain of salt, there is not really any way to know, history and science can give us good educated guesses, but they are probably wrong, alot.--155.144.40.31 (talk) 04:06, 14 August 2009 (UTC)[reply]
Surely you mean "This is why carbon dating has so little weight in historical circles dealing with documented history. It's useless when establishing the age of documents."? I wouldn't freely dismiss carbon dating for the eras before established civilisation where the mistake margin of a thousand years doesn't make such a drastic difference. TomorrowTime (talk) 06:26, 14 August 2009 (UTC)[reply]
Your account of Carbon dating inaccuracies doesn't seem to fully tally with that given on the Carbon dating article, and no reputable scholars doubt that we have manuscripts dating far earlier than 1200 A.D. (there are Egyptian papyri over three times as old). It's often impossible to date a manuscript exactly to a specific year (unless the scribe explicitly wrote down a date), but it's often quite easy to date a manuscript to a specific historical period (i.e. kingly reign, part of a century, etc.) -- and language scholars generally have no difficulties at all telling apart the English of 975 A.D. from the English of 1175 A.D. from the English of 1375 A.D... AnonMoos (talk) 07:03, 14 August 2009 (UTC)[reply]
So bottomline anything before 1500 AD for validity of New Testament copies should be taken "with a grain of salt." --LordGorval (talk) 12:13, 14 August 2009 (UTC)[reply]
No, that's not the bottom line at all. Where did that come from? Adam Bishop (talk) 12:56, 14 August 2009 (UTC)[reply]
No, that is not the "bottom line" of the accepted consensus of mainstream scholarship... AnonMoos (talk) 12:59, 14 August 2009 (UTC)[reply]
What source of book or manuscript was the Codex Sinaiticus copied from? 174.114.236.41 (talk) 20:50, 15 August 2009 (UTC)[reply]
That's unknowable at this date, but in many cases scholars can trace the development of the various textual traditions (Byzantine, Alexandrian, etc.) in great detail. See textus receptus, Alexandrian text-type, Byzantine text-type, Caesarean text-type, Western text-type ... AnonMoos (talk) 12:40, 16 August 2009 (UTC)[reply]


August 13

Danish princes and princesses

Are all male-line descendant of the Kings of Denmark given the title of Princes or Princesses? I mean even great-grandchildren and their descendants as long as they married lawfully even if not equally. What I am asking is if say Prince Henrik of Denmark had great-grandson and, everyone in the current royal family of Denmark have descendants too, would his descendants be entitled to the title of Prince and Princess? I asking this because I notice the Greek Royal Family are fourth and fifth generation descendants of Christian IX and I not sure how the Danish system of someone becoming prince works. Is it anything like the British system?--Queen Elizabeth II's Little Spy (talk) 00:25, 13 August 2009 (UTC)[reply]

I asked someone about that and I believe the answers was that all male-line descendants of the Danish monarchs are entitled to the princely title unless they forfeit it. I am not sure how it will work now when equal primogeniture is in effect. I mean, how equal is having children of a prince bear the princely title, while the children of his sister are not entitled to the princely title? It's just like styling the king's wife as queen and queen's husband as prince because the title of king is supposed to be higher than the title of queen - is that gender equality? Anyway, your question would've been answered much easier a year ago. Surtsicna (talk) 17:30, 13 August 2009 (UTC)[reply]
What language did Prince Philip, Duke of Edinburgh grow up speaking? Danish, Greek, English? Who then was a gentleman? (talk) 18:53, 13 August 2009 (UTC)[reply]
He was born on 10 June 1921, evacuated to France on 22 September 1922 and started at Cheam School in England in 1928 so would have been obliged to speak English from that point on. According to "H.R.H. The Story of Philip, Duke of Edinburgh" by James Hilton he never learned to speak Greek. -- Alexandr Dmitri (Александр Дмитрий) (talk) 03:43, 14 August 2009 (UTC)[reply]

Meles Zenawi's skin so tan

how come Meles Zenawi's skin color is not as black as he should. He is born in Ethopia. Is part of his family white? is it just images strangeness or he have some white family ancestory. Even his wife is darker than him.--69.226.34.249 (talk) 03:33, 13 August 2009 (UTC)[reply]

"Should"? Skin color is largely determined by melanin, and production of melanin is largely genetic. See the Human skin color article. Tempshill (talk) 05:14, 13 August 2009 (UTC)[reply]
Ethiopians in general do not have particularly dark skin tones. From pictures of Zelawi available on the web, his looks do not differ from those of average Ethiopians. --Xuxl (talk) 13:50, 13 August 2009 (UTC)[reply]
not all Ethopians is this tan. Azeb Mesfin is darker skin tone. Some ethopians is pitch black. Yes I know not all blacks is truly black.--69.226.34.249 (talk) 22:41, 13 August 2009 (UTC)[reply]

Satan, Lucifer and the Bible

Is it true that the Protestant-canonical books of the Bible don't support the idea that Satan is the fallen angel Lucifer? NeonMerlin 04:56, 13 August 2009 (UTC)[reply]

From our articles on Satan, Lucifer and Christian teaching about the Devil, it seems that whilst a "Lucifer" is mentioned in the bible, it's not explicitly said to be the devil. Early Christian authors connected the two, and this has continued to the present day. I recommend you go and read the articles I linked to, they are fairly in-depth. — QuantumEleven 12:15, 13 August 2009 (UTC)[reply]
There is some biblical evidence to back this up, but not much. Jesus says "I saw Satan fall like lightning from heaven" in Luke 10:18. The rest is, well 'doctrine added later'. DJ Clayworth (talk) 14:05, 13 August 2009 (UTC)[reply]
As it says near the beginning of the Satan article, the name Lucifer is sometimes used in Christian theology to refer to Satan, as a result of identifying the fallen "son of the dawn" of Isaiah 14:12 with the "accuser" of other passages in the Old Testament... AnonMoos (talk) 14:52, 13 August 2009 (UTC)[reply]

PRESIDENTIAL AND VICE PRESIDENTIAL SUCCESSION

IF BOTH THE PRESIDENT AND THE VICE PRESIDENT DIE IN A DISASTER, THE SPEAKER OF THE HOUSE BECOMES THE PRESIDENT. QUESTION: WHO BECOMES THE VICE PRESIDENT? —Preceding unsigned comment added by 98.225.97.156 (talk) 05:05, 13 August 2009 (UTC)[reply]

I assume you're asking about the US. See Twenty-fifth Amendment to the United States Constitution. Rckrone (talk) 05:10, 13 August 2009 (UTC)[reply]
(ec) 1. Please don't use all-caps. It's considered "shouting", and it's hard to read. 2. I assume you mean in the United States. The Twenty-fifth Amendment to the United States Constitution gives Congress the authority to determine who becomes President if both the President and VP are unable to serve; it has done so via the Presidential Succession Act. The 25th also states that vice-presidential vacancies are filled by the President and confirmed by Congress. So, the Speaker becomes prez per the P.S.A., and then the new Prez appoints a new VP, who must be confirmed by Congress. Tempshill (talk) 05:11, 13 August 2009 (UTC)[reply]
It should be noted that (partially) to avoid problems like this (but mainly for bigger reasons) most countries have a system whereby all heads of state or potential heads of state can't travel on the same plane/car/etc at the same time. Prokhorovka (talk) 09:05, 14 August 2009 (UTC)[reply]
Much like designated survivor -- the Cabinet member who gets to skip out of things like the State of the Union address. Apparently for Obama's inauguration, Secretary of Defense Robert Gates got to sit out. --- OtherDave (talk) 12:45, 14 August 2009 (UTC)[reply]

Less known cutting-edge authors from around the world

My usual literary diet tends towards western classics, but I’m interested in reading more contemporary art fiction. I’m looking for young (at heart), cutting-edge authors that maybe haven’t yet received overwhelming acclaim. Author who are artists, and who write with a higher goal than entertainment.

Could you suggest English language (or translated) authors who fulfill these criteria, from the following locations? (Feel free to list under headings.) Respectfully, --S.dedalus (talk) 08:41, 13 August 2009 (UTC)[reply]

You may want to look up Granta's recent issues entitled "Best Young British Novelists" and "Best Young American Novelists". Both are available from the magazine's web site or from a good library and contain examples of the work of around 20 authors from each country meeting your description. Although I should caution you that "cutting-edge" is often a synonym for "experimental", which may not be what you're looking for if your usual diet is classics. See a few more notes below of what could be called "future classics". --Xuxl (talk) 14:08, 13 August 2009 (UTC)[reply]
Thank you! I was looking for something like that. (I include Joyce as a "classic," so yeah, experimental is fine.) --S.dedalus (talk) 22:21, 13 August 2009 (UTC)[reply]

North America

Louis Hamelin and Jacques Poulin are two great contemporary novelists from Quebec. Hamelin deserves an article, by the way. --Xuxl (talk) 14:08, 13 August 2009 (UTC)[reply]

South/Central America

Great Britain

May I suggest David Devereux? --TammyMoet (talk) 09:20, 13 August 2009 (UTC)[reply]

Mainland Europe

Africa

A few good contemporary authors: Ahmadou Kourouma, Ngugi wa Thiong'o, Chimamanda Ngozi Adichie. Also check out the Egyptia Alaa Al Aswany. --Xuxl (talk) 14:08, 13 August 2009 (UTC)[reply]

Asia

A lot of good Chinese authors have been translated recently: Mo Yan, Jia Pingwa, Yan Lianke, Jiang Rong. Hitonari Tsuji is one of a large number of very interesting younger Japanese writers. In the Middle East, look up Fadhil Al Azzawi, Hoda Barakat, Ibrahim Nasrallah, Alia Mamdouh and in fact the entire American University of Cairo contempoorary Arab Writers series. Elif Shafak is a great young Turkish writer. --Xuxl (talk) 14:08, 13 August 2009 (UTC)[reply]

Not exactly lesser known but the japanese author Haruki Murakami is really popular both in Japan and in the west.--203.214.35.215 (talk) 13:05, 16 August 2009 (UTC)[reply]

Australia

Sri Lanka

I understand the LTTE was formed in response to instititional discrimination against Tamils - for example, the Sinhala Only act and different university admission criteria. The LTTE has been defeated but does Sri Lanka still practise such institutional discrimination? —Preceding unsigned comment added by 218.186.12.251 (talk) 09:23, 13 August 2009 (UTC)[reply]

I think that your question does not have a simple "yes or no" answer. I think that the Sri Lankan government would deny that it practices any form of discrimination toward Tamils; however, Sri Lanka has a long history of politicians from the dominant Sinhala majority demanding that the government adopt Sinhalese as the exclusive official language or Buddhism, the religion of the Sinhala, as the state religion (few Tamils are Buddhist; most are Hindu). This is evidence that some Sinhala at least do not believe that Tamils should be equal partners in Sri Lankan civic life. Many Tamil feel that they are treated as "second-class citizens" and subject to informal discrimination. This article has a good summary of the current situation. Marco polo (talk) 14:47, 13 August 2009 (UTC)[reply]
The Sri Lankan government dialed back some of the formal linguistic discrimination which formed the original early basis of Tamil grievances a while ago -- however, it was always too little too late from the point of view of preventing the slide towards civil war. And the basic political dynamic which mostly led to the past problems still remains -- namely, that that Tamils are in a permanent small minority in parliament, so that Sinhalese politicians can easily form governments without any Tamil participation, and therefore the various Sinhalese parties and political factions tend to compete to "outbid" each other in extreme and sometimes irresponsible Sinhalese nationalism in order to attract the most support from Sinhalese voters (not even bothering to appeal to Tamil voters)... AnonMoos (talk) 15:01, 13 August 2009 (UTC)[reply]

Blue Roofed Buildings

hi, after looking about on Google Earth i was just wandering why there are so many blue roofed buildings in places such as China and the Korean Peninsula, and what they are for?

thanks, --81.79.169.100 (talk) 11:14, 13 August 2009 (UTC)[reply]

I can't answer, but note Blue House. It appears to be a roof tiling preference, but what are antecedents?--Tagishsimon (talk) 11:23, 13 August 2009 (UTC)[reply]
Yes - what kind of "blue" is this? Could you show us a link for what you are referring to?
If it's a darkish ceramic grey-blue, that's roof tiles. If it's a bright blue, that could be corrugated sheet metal roofing, or tarps. --PalaceGuard008 (Talk) 04:12, 14 August 2009 (UTC)[reply]

The most obvious example is the Temple of Heaven in Beijing, and its near-mirror in Taipei. DOR (HK) (talk) 06:04, 14 August 2009 (UTC)[reply]

Are you referring to the Chiang Kai-shek Memorial Hall? --PalaceGuard008 (Talk) 07:21, 14 August 2009 (UTC)[reply]

Allied Bombing in WWII

How many of people attributed to being killed in the Hollacaust durring WWII were killed by Allied Bombing of factories and transportation work-crews staffed by slave labor ? ≈≈≈≈≈ —Preceding unsigned comment added by 24.62.6.182 (talk) 14:43, 13 August 2009 (UTC)[reply]

The highest estimate of all German/European civilian casualties of Allied strategic bombing in WWII is 600,000 - see Strategic bombing during World War II - as compared to the generally accepted estimate of 6 million deaths in the Holocaust. Also, the extermination camps, where about half of the Holocaust victims died, were deliberately located away from major population centres. So Allied bombing casualties cannot have been a significant factor in the Holocaust. Gandalf61 (talk) 15:22, 13 August 2009 (UTC)[reply]
Furthermore, I think that the term Holocaust refers to the deliberate mass murder of Jews by the Nazi Germans, not accidental deaths due to other causes during World War II. So I think that the answer to the original question is zero. Marco polo (talk) 16:02, 13 August 2009 (UTC)[reply]
The Holocaust in general refers to Jews, but I do believe that other groups such as Gypsies and homosexuals are included in the count, though I could be wrong about that. But forced labor in factories and transportation work crews was not done by Jews so much as by kidnapped Slavs, etc. See Forced labor in Germany during World War II. Who then was a gentleman? (talk) 18:57, 13 August 2009 (UTC)[reply]
6 million jews, 11 to 17 million if you count everyone, according to the article. TastyCakes (talk) 19:31, 13 August 2009 (UTC)[reply]
I would not call the deaths of civilians resulting from bombing of cities "accidental." Besides bombing of military installations and armaments factories there was terror bombing. Terror bombing was an Allied policy, to try and break the will of the German and Japanese people to continue the war. Edison (talk) 19:15, 13 August 2009 (UTC)[reply]
As opposed to the non-terror bombing used by the Germans a few years earlier. Googlemeister (talk) 20:04, 13 August 2009 (UTC)[reply]
I agree that the deaths of German civilians as a result of Allied bombing were not accidental. However, the Allied bombers did not intend to cause the deaths of captive Jews, Gypsies, and others, so those deaths were accidental. The issue here is whether Jews (or Gypsies, or homosexuals, or Slavs) killed by Allied bombs are included in the numbers of those killed in the Holocaust. The Holocaust was the deliberate murder of these groups by the Nazi Germans, so Allied bombing was not part of the Holocaust. The numbers for the Holocaust are estimates with a reasonable degree of accuracy, but the numbers (of Jews, Gypsies, homosexuals, etc.) incidentally killed by Allied bombing are probably within the estimates' margins of error. According to a strict definition of the Holocaust, however, those deaths should not be included, since they were not deliberate killings by Germans or their allies. Marco polo (talk) 20:32, 13 August 2009 (UTC)[reply]
Accidental is an unforunate word in this context - incidental might be better. --PalaceGuard008 (Talk) 11:54, 16 August 2009 (UTC)[reply]
The best answer I can give is "Not many proportionately, but at least some 400 in one incident alone". On 24 August 1944, an American air raid on the Wilhelm-Gustloff-Works, an armaments factory inside Buchenwald camp, also struck nearby I-Barracks, and an adjoining temporary structure. Many of the prisoners housed there died almost immediately, while others died later from their wounds. Among the dead were Dr Rudolf Breitscheid, who had been a Social Democratic deputy in the Reichstag, and Princess Mafalda of Savoy, wife of Prince Philipp of Hesse-Cassel. (Source: Royals and the Reich, Jonathan Petropolous, 2006; Petropolous in turn cites The Buchenwald Report, edited by David Hackett, 1995.) AlexTiefling (talk) 20:28, 13 August 2009 (UTC)[reply]

Polity IV and Freedom House

The Polity IV and Freedom House measures of democracy are highly correlated. Which point on the Polity scale is roughly equivalent of Freedom House's "electoral democracy"? Someone should have done research on this, but I cannot find any. Jacob Lundberg (talk) 19:32, 13 August 2009 (UTC)[reply]

Why does Paul Biya look so young

Paul Biya don't even look differnt since the 1960s. He is 76 right now and he don't even look 35. Even some 30 year-olds look older than him? Why is that he look so younger, even looking younger than Jakaya Kikwete.--69.226.34.249 (talk) 22:57, 13 August 2009 (UTC)[reply]

See The Picture of Dorian Gray. --// BL \\ (talk) 23:58, 13 August 2009 (UTC)[reply]
Having never heard of the gentleman before (and therefore having no preconceptions about him), in the linked picture he looks to my (52 y-o European) eyes to be in his late 50s or early 60s. He certainly does not look as old as 76 (I assume the picture is contemporary), but apparent ages can be greatly influenced by different (inherited) bone structures and complexions; did his parents look younger than their actual ages? He may also have avoided the skin-ageing effects of such things as smoking and undue exposure to sunlight. Amusingly, exactly a year ago a couple of new acquaintences in their 20's estimated my own age as 35, but admittedly we were in a poorly-lit pub at night. 87.81.230.195 (talk) 00:38, 14 August 2009 (UTC)[reply]


Because black don't crack? Adam Bishop (talk) 01:29, 14 August 2009 (UTC)[reply]
Just wondering if the OP posted the correct link, because I think the link is actually for Janet Museveni. The question did made me curious though and a quick Google search yield this - a photo of Paul Biya in Paris on July 14, 2009 from Daylife. I have to agree with the OP he does look 30 ~ 40ish. Maybe the effect from some sort of secret anti-aging treatment/drug reserve only for Cameroon President? Royor (talk) 07:05, 14 August 2009 (UTC)[reply]
There are some pictures of Biya (along with his notably-coiffed First Lady) further down. Speaking of African leaders who don't look their age, Robert Mugabe could easily pass for a man in his 40s, while he's actually 84. --Sean 14:22, 14 August 2009 (UTC)[reply]
From the Paul Biya article

The historian David Wallechinsky, in his book Tyrants, the World's 20 Worst Living Dictators, ranked Biya with three others in sub-Saharan Africa: Robert Mugabe of Zimbabwe, Teodoro Obiang Nguema Mbasogo of Equatorial Guinea and King Mswati of Swaziland.

With the exception of King Mswati, all three sub-Saharan Africian leaders don't look their age. Coincidence? I don't think so - it got to be some sort of secret anti-aging mineral/drug/plant/voodoo magic found only in sub-Saharan Africa! That must also be the reason why they are all so keen on holding on to their power there too. XD Royor (talk) 15:45, 14 August 2009 (UTC)[reply]
Usually black people don't look their age. In General none of those African genocides look their age. No, Paul Biya don't even look 40s even clear pictures. None of his hairs is gray or white. One poster who said 3 worst dictators, Paul Biya isn't that bad ruler. I have no idea by what's a worst dicators. The equator guinea guy i don't know him. I've hear Abdoulaye Wade and John Kufuor won gold prize, they are known as best buddies. i think Paul Biya and Robert Mugabe is just excellence, this is why I think they stubborn on hopding their powers. I don't think tose 3 guys is that bad in power. Omar Bongo is also a long lasting guy, but he die two month ago. Daniel Arap Moi I think is a worst dictor. in 1982, Raila Odinga try to kick Moi out of power. It is all people's opinion base on the rulers of powers.--69.229.39.33 (talk) 20:44, 14 August 2009 (UTC)[reply]
Heh...Wallechinsky is hardly an historian. He just makes lists that are even more poorly referenced than Wikipedia. Adam Bishop (talk) 17:30, 14 August 2009 (UTC)[reply]
In every clear picture I've seen of him, he does not look that young. He looks 70ish. It shows through. Vranak (talk) 15:53, 14 August 2009 (UTC)[reply]
Sorry if it is misleading but I was just quoting the article, and I have no knowledge whatsoever in the topic of African dictators. Vranak do you have a link to clear picture of Mr.Biya? Royor (talk) 14:41, 15 August 2009 (UTC)[reply]

More on US presedential succession

The above all caps question made me think a bit, and I wondered: Who would succeed to the US presidency should everyone in the line of succession be killed in one event, such as a nuclear attack on Washington DC (or an assault on an event where everyone in the line was present, such as the State of the Union address)? Does the government take any steps to prevent such an occurrence? Ks0stm (TC) 23:21, 13 August 2009 (UTC)[reply]

Designated survivor has some info. Algebraist 23:33, 13 August 2009 (UTC)[reply]
Short answer: yes, they take steps, by always having one person in the line somewhere else when events like that take place. They have been doing this since the Cold War (probably since the early 1950s, actually, I haven't found any evidence of them using it before the 1980s, which is kind of odd, but does coincide with decreased warning times of potential attacks). --98.217.14.211 (talk) 00:02, 14 August 2009 (UTC)[reply]
IThe Secretary of Agriculture or some such constitutional/statutory successor would be always be away from Washington during the inauguration. If they too were snuffed, then the various Governors could appoint replacement Senators, and the new President Pro Tem of the newly appointed Senate could become President. House elections would take place eventually, after several months as state laws decree and the U.S. government would be reconstituted. Edison (talk) 03:30, 14 August 2009 (UTC)[reply]
I believe then it would be time for President Starkey to assume his position. TomorrowTime (talk) 06:34, 14 August 2009 (UTC)[reply]
Short answer is that you would have a constitutional crisis and all bets would be off. Googlemeister (talk) 13:21, 14 August 2009 (UTC)[reply]
I agree. It's incredibly naive to assume that if the US government were decapitated that the military commanders who would be running the response would all roll over for a civilian far down the line of succession. Here's what General Tommy Franks had to say on what would occur: "[an attack could cause] our population to question our own Constitution and to begin to militarize our country in order to avoid a repeat of another mass, casualty-producing event". --Sean 14:37, 14 August 2009 (UTC)[reply]
It might be tough for some general to convince all the other generals and admirals that the U.S. should abandon 220 years of democracy and government by elected civilians just to further his ambition to rule. Interservice rivalry would be a problem, just like separation of powers. This is about a decapitation, not the general downfall of civilization by some attack that wiped out most of the population and totally disrupted civilization. Edison (talk) 15:11, 14 August 2009 (UTC)[reply]
True, but it is a situation the rules (Constitution) has not fully contemplated, and in a "crisis" it is a lot easier to make up new rules. Googlemeister (talk) 15:36, 14 August 2009 (UTC)[reply]


August 14

What sort of photographic processes are these?

The history of photography and list of photographic processes articles are kind of overwhelming, so maybe someone here can help me... I'm going through old family photos and some are super old. One is made of metal and is probably some sort of daguerrotypeortintype or something. When did these sorts of metal photographs fall out of common use? Another photo is somehow embedded in glass, or is itself glass. What type of photograph is this, and when was it made? Calliopejen1 (talk) 06:35, 14 August 2009 (UTC)[reply]

From your descriptions, sounds like a tintype and an ambrotype. Compare them to the illustrations in the articles. Ambrotypes were in the decade spanning the 1850s and 1860s, and tintypes replaced them in the 1860s.- Nunh-huh 07:35, 14 August 2009 (UTC)[reply]

Best investment

Which of this investing option are best strategies either invest in money market, fixed income, structured products or equities. please give recomendations.ty. —Preceding unsigned comment added by 202.188.130.36 (talk) 08:59, 14 August 2009 (UTC)[reply]

Hopefully someone will be able to give information on the different strategies and the state they're currently in, but firstly, under our General and risk disclaimers, I'd recommend you don't take financial advice from Wikipedia. AlmostReadytoFly (talk) 10:41, 14 August 2009 (UTC)[reply]
You can find more information by reading our articles on money market, fixed income, structured product, equities and their linked articles. The "best" investment strategy will depend on whether you want to invest a lum sum or make regular deposits; how much you can invest; how long you can invest it for; how liquid you want your investment to be; how much risk you want to take on; what level of income you want to receive; what overall yield you want to receive; your tax status; what other investments you have and probably twenty other factors. So your question is like asking "which is the best colour" - there is no single answer. A good rule of thumb is to assume that free financial advice is worth what you pay for it. Gandalf61 (talk) 10:57, 14 August 2009 (UTC)[reply]
I agree that the answer depends on each individual's circumstances and assumptions. However, virtually all financial advisors agree that diversification is a crucial part of any investment strategy. So you should be sure to spread your investments among asset classes. Marco polo (talk) 14:27, 14 August 2009 (UTC)[reply]
I'm in the same boat with 202 and wondering along similar lines. Is there any study/data on investment pattern/strategies/outcome for say, the past 30 years for say, the US/CAD population (or maybe in a certain annual income range)? I tried to google for it but I got mostly for/against investment in education and/or health care. Royor (talk) 16:01, 14 August 2009 (UTC)[reply]
I think The Intelligent Investor is a good place to start for a sensible take on investing. It looks at a lot of data (I think going back a hundred years in some cases), but keep in mind the last edition was published in 1971. There is a reprint, with commentary by Jason Zweig that tries to tie it in with modern examples. TastyCakes (talk) 16:09, 14 August 2009 (UTC)[reply]
Warren Buffett and I agree with TastyCakes. Read that book. Tempshill (talk) 19:00, 14 August 2009 (UTC)[reply]
The best strategy huh? That depends -- do you want to lose money, make money, or keep yourself occupied? The only sure strategy I know of is something akin to Warren Buffett's: identify a quality product that is appearing on new markets. For instance, about seven years ago I noticed how good Pepsi tasted. I figured there would be major growth in Asia. So if I had sunk $10,000 into PepsiCo I'd probably be looking at $12,000 ten years later. Not a very exciting profit. This is why I don't invest. Vranak (talk) 21:36, 14 August 2009 (UTC)[reply]
The problem is that your observation that Pepsi was tasty and was ready to expand into new markets was a fairly obvious one, and had therefore been factored into the stock price by the market. I'm sure Buffet would be the first to tell you - just because a company is great, doesn't mean it's a good stock to buy at any price. TastyCakes (talk) 22:27, 14 August 2009 (UTC)[reply]
Absolutely. Two other points then:
1. Your perception should be more astute than that of potential or actual investors in PepsiCo
You need to know that the Chinese will like Pepsi more than Coke, but others must believe the opposite. You can think of like this: you're taking the Coke backers' money, even if the complexities of the market make this 'theft' 'not that simple'.
2. Your perception must be congruent with that of an emerging market
(i.e. The Chinese need to agree with you that Pepsi tastes good).
Vranak (talk) 23:33, 14 August 2009 (UTC)[reply]
I agree the book is good but I'm wondering if there are other similar study/data other than the book. For example: This is the Average household expenditures of a typical Edmonton/Vancouver family in 2007. I'm guessing investment would fall under Personal insurance payments and pension contributions (which is CAD$3810 for Vancouver). I tried but couldn't find further breakdown (what people did using this $3810). It doesn't even have to be a study, even just a very general raw data on what people had done would be great. Royor (talk) 14:56, 15 August 2009 (UTC)[reply]

Any professional financial adviser worth listening to would refuse to answer the OP's question. First, there is no information on the client; second, there is no information on the client's current financial situation; and third, there is no information on the client's risk profile. All of the above advice, if given by a professional, would be illegal in many jurisdictions. DOR (HK) (talk) 04:01, 17 August 2009 (UTC)[reply]

Perhaps, but how much of the "risk assessment" is actually any good anyway? When you go to a Canadian bank to buy mutual funds, they will put you through a test to supposedly decide your "risk tolerance" (based on age, financial need, general bullishness etc) and what mutual funds you should buy as a result. Yet if you look at the histories of those funds that story doesn't really play out - with the exception of the "super safe funds" (which you can be assured will trundle along at somewhere close to the inflation rate) they tend to be all over the map, and many if not most do not outperform the index. On top of that you pay near-extortionist rates for a lot of these funds - over 3% in many cases. I first went to a bank to buy mutual funds when I was in my third year or university or so, and not knowing any better let the "financial adviser" (a young guy probably a couple years out of some unrelated arts degree) sell me on this computer designed portfolio business. What seems clear to me now is that the bank was not particularly interested in me making the soundest investment choices but using their products. Had they been more interested in the former, they should have pointed me to low fee index funds and bond funds, an area my bank (BMO) is not particularly good at. The only way to properly invest (in my opinion) is to educate yourself - you cannot depend on the bank telling you the best thing to do. Now I'm not saying people shouldn't listen to so called financial experts, bank supplied or otherwise. But they should not depend entirely on one source, particularly not one with a vested interest in you buying their product. TastyCakes (talk) 19:20, 17 August 2009 (UTC)[reply]

Cross-party voting in US federal elections

Is there any data about voting for candidates not in the voter's party in US federal elections? Such as whether Republicans or Democrats tend to do it more, and if it's done more often for Representative, Senate or Presidential candidates. Thanks. ÷seresin 09:19, 14 August 2009 (UTC)[reply]

This may be a naive question, from a Brit, but isn't a voter's party simply the party whose candidate the voter votes for? And as it's a more-or-less secret ballot, how could there be any reliable data on who voted for whom anyway? AlexTiefling (talk) 11:05, 14 August 2009 (UTC)[reply]
From voter registration: In some states, when registering to vote, one may declare an affiliation with a political party. This declaration of affiliation does not cost any money, and it is not the same as being a dues-paying member of a party; for example, a party cannot prevent anybody from declaring his or her affiliation with them, but it can refuse requests for full membership. Some states, including Michigan, Virginia, and Washington do not have party affiliation with registration.
Perhaps exit polls could give data on who voted for whom. AlmostReadytoFly (talk) 11:19, 14 August 2009 (UTC)[reply]

If the show Blackadder has any truth, it seems that it was tradition for candidates/prospective MPs to not vote for themselves at elections - i.e. gift their vote to an opponent. Not sure if that's true thoguh - Blackadder likes to mix reasonably accurate history with silliness and made up stuff! 194.221.133.226 (talk) 11:28, 14 August 2009 (UTC)[reply]

CNN exit polls for the 2006 House and 2004 Presidential elections give breakdown by party. It's not much to go on though. (via). AlmostReadytoFly (talk) 11:38, 14 August 2009 (UTC)[reply]
A tangent related to primaries and party registration:
In states with closed primaries (e.g., Maryland, where I've served as an election judge), you can vote in a party's primary only if you're registered--with the board of elections, not with the party itself. (The Maryland voter registration form allows you to "affiliate" with any of six recognized parties.) That's because the closed primary is how the party chooses its candidate who will run in the general election.
This means that you must be a registered Republican to vote in the Republican primary. You can, of course, change your party affiliation at any time; if you do so before the cutoff for the primary, you can switch from one party in order to vote in its primary, then switch back later.
Independent ("unaffiliated") voters can cast ballots in the primary only for non-partisan offices, if there are any (local school board, county sheriff, etc.).
Party affiliation means nothing in terms of voting in the general election. Regardless of registration, you can vote for whatever candidate you please.
--- OtherDave (talk) 13:01, 14 August 2009 (UTC)[reply]
Where you're most likely to get crossover in the U.S. primaries is either as a protest against your own party, or more likely, to deliberately vote for a "weaker" candidate in the opposite party. Which is really only effective when done en masse, of course. Baseball Bugs What's up, Doc? carrots 13:07, 14 August 2009 (UTC)[reply]
A perhaps more common case (since it's less cunning and dependent on mass-crossover) is in districts that are safely Republican or Democratic, the primary essentially decides the winner. If you want to have an effect on the outcome, your only chance to do so is in the primary. --Sean 14:58, 14 August 2009 (UTC)[reply]
In the U.S. when the President is an incumbent up for reelection, from your own party, and is virtually assured of the nomination, there is temptationto crossover and vote for the idiot/clown candidate from the other party, so he/she will be easy to defeat. But the rub is, Mr/Ms Idiot might actually win and then you would have four years of the country lurching around with idiotic policies which have a harmful effect on the entire world. It can take a long time to recover from four or eight years of idiot/clown administration. Edison (talk) 15:05, 14 August 2009 (UTC)[reply]
Luckily, that almost never happens in the U.S. Baseball Bugs What's up, Doc? carrots 15:19, 14 August 2009 (UTC)[reply]
Yes, there is plenty of data of the kind you're asking about. They are based on polling data (since actual votes are secret) but are usually pretty accurate. Fivethirtyeight.com has tons of links to all kinds of polls every day. 70.90.174.101 (talk) 05:48, 15 August 2009 (UTC)[reply]

Keeping the information from AlmostReadytoFly in mind, the basic question is how voters define themselves. In paeticular at what level does one voye more for the party and less for the candidate?

So, the short answer to your question, US voters are most likely to stray from their normal party in voting for the President. Of course, that might also be the harbinger of a long-term change in affiliation. B00P (talk) 08:27, 15 August 2009 (UTC)[reply]

Opposition to health care reform in the US

Here in the UK, the media (notably the BBC) is pretty aghast at the criticism levelled at the NHS by the opponents to health care reform in the US. What they haven't explained is why there is so much criticism of President Obama's plans? For me it seems to be a no-brainer that reforming the system of expensive insurance that leaves millions of American without access to affordable health care is something that is long overdue. Astronaut (talk) 15:40, 14 August 2009 (UTC)[reply]

Well being in Canada, maybe I shouldn't comment, but I think the bottom line is that health care does work for the majority of Americans (particularly the ones that pay attention to what's going on around them and, critically, actually vote), despite it being a very cost-ineffective system, and people are opposed to change based on fear of losing what they have, perhaps most specifically of losing control of their health care to the government (whatever that means) and having to endure longer waiting periods (as in Canada). However, the health care problems have been getting more severe in recent years - specifically, more and more people are becoming uninsured or under-insured as prices escalate - so I think most people are open to change in the system. There is just huge disagreement on how best to do that. TastyCakes (talk) 16:17, 14 August 2009 (UTC)[reply]
The following is not my opinion. It is a very generalized synopsis of complaints about nationalized healthcare in the United States that probably isn't given airtime in foreign press:
  • The U.S. does have government run free healthcare for the military. It is considered by many to be terrible.
  • There are many free clinics throughout the United States that offer free health care on demand. The fear is that these will be shut down and replaced with government run clinics that require a long waiting period for care.
  • Many people do not have insurance because they don't want insurance. That means that the number of uninsured people who want insurance is far lower than what is reported.
  • This healthcare will not be free. Instead of paying for insurance and then having the insurance company pay the health care bills, we will pay higher taxes and then have the government pay the health care bills. Experience has shown that the United States government cannot be trusted with handling money.
  • Finally, there is the counter-argument to the "Socialized medicine is better". At any point in time, there are people from countries with great socialized medicine programs coming to the United States for care. Why would they come to a country with worse health care?
I hope you can see that the opposition to nationalized health care isn't based purely on stupidity. Some people have somewhat valid reasons for their opposition. -- kainaw 16:19, 14 August 2009 (UTC)[reply]
There are a lot of issues that are clouding the water. One of the biggies is that the government plan will make other insurance non-competative and that employers will cancel the other insurance plans in favor of the government option. Many Americans that I personally know have doubts over whether the government can actually operate this better then the existing system. That and it costs $1,000,000,000,000.00. Googlemeister (talk) 16:21, 14 August 2009 (UTC)[reply]
Further, I think a lot of people believe some of the often cited problems with the current system (defensive testing due to malpractice insurance, lack of emphasis on preventative measures, high drug prices etc) can be solved without nationalizing the whole thing. TastyCakes (talk) 16:25, 14 August 2009 (UTC)[reply]
One trillion, zero hundred zeroty-zero billion, zero hundred zeroty-zero million, zero hundred zeroty-zero thousand, zero hundred zeroty-zero dollars and no cents? Grumble. Every one of those zeros is most likely wrong. —Tamfang (talk) 19:59, 19 August 2009 (UTC) [reply]
Much of the difference is rhetorical.
Here in the UK, it's common to talk about the NHS and associated services as 'public healthcare'. The use of 'public' evokes the idea that the service is of the people, by the people, for the people; 'healthcare' suggests the idea of something supportive, and targeted towards upholding people's general health. And attitudes reflect this - I regularly go to my doctor for a check-up, because I know it won't cost me any more than not doing so, and I want to have my general level of health upheld.
In the US (where I have travelled extensively and spoken to quite a range of people on this topic), the discussion is framed in terms of 'socialized medicine', a phrase which most British people would not have recognised before the past week's news reporting. By calling it 'socialized', there is an obvious comparison drawn with socialist systems in the stricter sense, such as those of Cold War era Eastern Europe. Unlike the UK's current system (or Sweden's, for that matter), those systems did epitomise much of what was bad about socialist or (nominally) communist government from the point of view of a typical citizen. But beyond that, and less cynically, 'socialized' also brings up the idea of the individual's safety being entrusted to a corporate entity, which will not know the patient's mind or feelings as well as the patient does personally. In an environment where (of one if sufficiently affluent) one has reasonably free choice over one's treatment, sacrificing some of that choice will inevitably look unappealing. And by saying 'medecine' rather than 'healthcare', the product, rather than the outcome, is emphasised, leading to a focus on the commodity value of individual treatments rather than the holistic effect of the system.AlexTiefling (talk) 16:42, 14 August 2009 (UTC)[reply]
Part of the problem with the media (in Britain I assume as much as in Canada) is that we only normally see the most ridiculous protesters, because it makes the news more entertaining. So, most people don't really believe there will be death squads coming to execute your grandparents, or that this is a communist plot; at least, I have to assure myself that people don't really believe this, because I don't think I would be able to live on the same planet as them. Anyway, "free universal health care" in Canada is pretty cool, but it's not really free, and it's not totally universal; I can go to the doctor for a checkup, and I can go to the emergency room in the hospital, and I can give them my OHIP card and that's that. But sometimes government health insurance doesn't cover everything, especially if you want small luxuries like a private hospital room; and sometimes you can be covered for things by private insurance, or at least insurance offered by your employer (which is of course paid for out of your paycheque), that either are not covered by government insurance or are just covered better by private insurance. There are rare cases where someone's life is ruined because they have to pay for health care in Canada, but for the vast majority of people it's not something you ever notice or think about, it's just there and it is apparently free. Adam Bishop (talk) 17:12, 14 August 2009 (UTC)[reply]
It is mostly a fear of changing something, mixed up with half-baked rhetoric about socialism and democracy and things like that. It is a far less informed and rational debate than one would hope, for such an important issue. There is a large camp of "oh, the status quo sucks, but I haven't seen anything better," which will never result in anything being done. And there is a huge, huge, huge amount of scare-mongering, misrepresentation, and out-right ignorance. --98.217.14.211 (talk) 17:22, 14 August 2009 (UTC)[reply]
If you want some more reading OP, we have an article with some good sources: Health care reform debate in the United States Livewireo (talk) 17:33, 14 August 2009 (UTC)[reply]
Actually, "President Obama's plans" are not a Canadian-style or UK-style government-paid or government-run health care system. No voter has read the America's Affordable Health Choices Act of 2009, which was recently passed by the U.S. House of Representatives; it is over 1000 pages in length. (I figure no Congressperson has read it either, for that matter.) Additional legislation is planned but there will be fatigue as time goes on and IMO the probability of success for each piece of legislation will drop as the next two bills are introduced. Obama himself confused me personally a couple days ago with his off-the-cuff answer to a reporter's question where he stated that Canadian-style health care would not work in the US. (Errr, why is that?) I figure that 99% of the public does not know what the health care bill entails. In general the USA has a history of claiming to dislike the government and bitching about high taxes (while simultaneously actually growing the size and role of government over time, even in the Reagan era). The protesters are of those who claim to dislike an increased government role in society in general. They think that they will pay a lot of money and get little or nothing in return — the "Harry and Louise"-style complaint is that there will be bureaucracy and "a government bureaucrat will decide whether you get a hip transplant". (This disregarding the fact that currently it's a corporate bureaucrat who decides this.) There is also a cold element, IMO, related to the American Dream that "worthy" people who work hard become prosperous; those who are not prosperous are not hard workers or worthy, so why should the hard workers pay for their health care? Tempshill (talk) 17:45, 14 August 2009 (UTC)[reply]
(ec) If people understand the difference between liberalism in US (which Obama ascribes to) and liberalism in Europe (see the liberalism article, the criticisms may be more understood. The American liberalism is very egalitarian, emphasizing equality and regulation, where conservatism places a greater emphasis on personal freedoms and privatization (and therefore less government regulation). The two ideologies are often presented as if they are mutually exclusive. Even though there has been an increase in liberalism in the past few years (hence Obama's victory), there is still a large number of Americans that are unaccepting of that ideology, and therefore hostile to these types of reform. —Akrabbimtalk 17:51, 14 August 2009 (UTC)[reply]
Of course the BBC would be aghast over criticisms of NHS, BBC is a government network! Wrad (talk) 18:18, 14 August 2009 (UTC)[reply]
That's a misleading way of thinking about it. The government does not generally exert editorial control over the BBC (and the times when it has have been infamous). To quote the article: "The BBC is required by its charter to be free from both political and commercial influence and to answer only to its viewers and listeners." It might be fairer to say Britons are aghast at the criticism out of national pride and because much of it isn't true. Note how David Cameron frequently has to reassure the public that the Tories won't harm the NHS.
POV: Much of the criticism of the BBC's alleged (and not entirely fictitious) partiality comes from strongly right-wing media, including competitors with a vested interest in its failure. AlmostReadytoFly (talk) 18:42, 14 August 2009 (UTC)[reply]
A lot of this sort of stuff happened in Europe, fifty or sixty years ago it used to be like the states. Doctors campaigned against the state interfering in their work and poor people died after they had sold everything. Bishops argued against it saying it was creeping communism and an affront to God. Politicians flung mud at each other. Even today there's real problems with dentistry. This all seems to just be a rerun. Dmcq (talk) 19:34, 14 August 2009 (UTC)[reply]
Americans would be pretty ticked if the government tried to take over the news. It's illegal for the government to broadcast Al Hurra, for example, within the US borders. It's an entirely different world over here. People don't consider themselves "subjects" of any government or any person, and never have, and for centuries have bristled at the very idea (see events surrounding the US flag in the 1908 Olympics in London. US conservatives don't like big government, don't trust it, and don't want it taking over the press, the health system, or anything else of the sort. Americans have been this way for a long time. Nothing new under the sun here. Wrad (talk) 19:37, 14 August 2009 (UTC)[reply]
What about NPR? I think it's a stretch to say the BBC has "taken over" the news, it's not like there aren't other news sources, and in case you missed it the first time, it's written into its charter that the BBC is supposed to be independent. And I think they do a pretty good job of being fair, at least as good as the major American outlets. The coverage seems to me much broader geographically than American channels and the overall quality of the reporting is high, in my opinion. I'm not sure most Americans would object to it on the ideological grounds you cite, but rather the having to pay the license fee. TastyCakes (talk) 19:50, 14 August 2009 (UTC)[reply]
NPR is not even close to BBC. If you look at the funding section of it's article, you will see that only about 2% of its funding come from government grants. The rest is private donations and fees charged to member stations. Wrad (talk) 20:06, 14 August 2009 (UTC)[reply]
While I won't disagree that the funding is very different, do you not agree that their news programs come from similar philosophies? That philosophy being to provide independent, non commercial news to citizens, with government support (which according to the article, accounted for the majority of NPRs funding in earlier decades). It was my understanding of your previous post that you think such a philosophy doesn't exist within the US, but I believe it does and that the NPR (and PBS) represent that, although in recent memory they have been publicly funded at a fraction of what the BBC receives (so I would grant you that the mantra appears stronger in the UK than the US). TastyCakes (talk) 20:43, 14 August 2009 (UTC)[reply]
The only thing the conservatives in the US seem comfortable letting the government run is the military. Googlemeister (talk) 19:43, 14 August 2009 (UTC)[reply]
And even there they've been big fans of privatization. --98.217.14.211 (talk) 19:55, 15 August 2009 (UTC)[reply]
Astronaut asks "why there is so much criticism of President Obama's plans?" Of course any response to a why question, especially about current events, will bring a POV answer. Here is mine: In place of the nonprofit, government-regulated health care system that most industrial nations have, the United States has a privatized health care industry, with several sectors reaping rich profits from it. There is the insurance sector and the pharmaceutical sector, there are doctors and hospitals, all free to charge people, some facing a choice between life and death, whatever they think they can get. Hundreds of thousands of very affluent people in the United States, comprising a substantial portion of the moneyed class, derive their rich incomes from this industry. Obama's plan, by setting up a government-run insurance program that would have the power to negotiate costs, threatens those rich incomes. Consequently, those rich people and their companies and industry associations, and the media personalities and politicians beholden to them (for example, for contributions to finance their political campaigns) are advancing a vigorous campaign against Obama's plan, often involving outright lies. The US media may be reluctant to question these distortions and lies for fear of losing advertising revenue from the industries advancing those distortions and lies.
I also want to respond to TastyCake's claim that healthcare works for the majority of Americans. I think that there are conservatives making that claim in Canada in the hope of opening up private investment opportunities in healthcare in Canada, but I can testify, as an American with one of the better healthcare plans available, that it does not work so well. First, I face long waits for an appointment with a doctor, often more than a month for my own "primary care physician". I need to see the primary care physician before I can be approved to see a specialist, again after waiting for many months for the next available appointment. Then, if my physician or specialist wants to order a procedure, I need to request my insurer's approval. Fortunately, I haven't needed any serious procedures, but there are nightmare stories out there about insurers denying approval for needed procedures (to protect their profits, of course). Finally, and most serious of all, even this problematic access to healthcare will disappear for me completely if I lose my job, or if I dare to leave my job. (Many Americans are shackled to jobs they hate because it is their only access to healthcare.) Of course, not every job comes with insurance. Without job-based insurance, Americans, unless they are in extreme poverty and qualify for government-funded Medicaid, face astronomical bills for healthcare. For example, an office visit with a doctor can be $250. Simple operations can be $8,000. More serious treatments involving overnight stays in hospitals can cost $50,000 to $100,000 or more. If you are very rich, to the point that tens of thousands of dollars are trifling expenses, you can get very good healthcare in the United States with little trouble. But for the vast majority, the system does not work well. Marco polo (talk) 19:46, 14 August 2009 (UTC)[reply]
I think Marco polo's answer is most to the point, actually — the why is indeed because a lot of affluent and wealthy people will see their incomes be reduced, and all the arguments against more involvement of the US government in health care are just fronts. The money is the real reason that you see a lot of these disagreements. Tempshill (talk) 20:52, 14 August 2009 (UTC)[reply]
The US healthcare biz is wildly profitable. They spend 1.4 million dollars every day on lobbying US lawmakers. They are not going to give up their cash cow without a brutal fight. This guy Wendell Potter (former healthcare flak who grew a conscience) is pretty interesting: [2]. --Sean 21:13, 14 August 2009 (UTC)[reply]
I guess it all comes down to this : "State, bad ; Market, good". The rest is just a way to fill up the stereotype (and there are huge vested interests that are ready to feed the true believers with whatever they want to hear). No one really cares about the basic facts : the US spends 18% of their GDP on health care, 1 us resident out of 6 is without insurance. As a comparison, France spends only 11% of its GDP on healthcare, everyone is insured by the "sécurité sociale", even people who have just lived for 6 mouths in France, and we enjoy a longer life expectancy. By the way, the『sécurité sociale』has nothing to do with the British NHS : there are many ways to organise a public health system. The French one is much more market oriented. Gede (talk) 00:37, 16 August 2009 (UTC)[reply]

I think TastyCakes (mmm...Tastykakes) has it best. Fifteen percent of Americans lack health insurance. That's an extraordinary figure. But it also means that 85% of Americans do have health insurance. And surveys show most Americans like their health-insurance plans. So most people may have something to lose through healthcare reform. America actually has an excellent healthcare system for people who have good health insurance, can afford their copayments and premiums and don't acquire a horrible disease that ramps up their out-of-pocket expenses. In America, there is no doctor shortage and short waits to see specialists. The hospitals there actually advertise to try to attract patients. Since people tend to vote with self-interest in mind, proponents of healthcare reform have to convince the majority of people, who have health insurance and like it, what's in it for them. -- Mwalcoff (talk) 22:24, 16 August 2009 (UTC)[reply]

I think you’re all joyously missing the point. No one in US politics or (especially) the media cares one bit whether something offends Brits, Canadians or anyone else. Why are some people bad-mouthing healthcare services elsewhere? Because they can’t come up with purely domestic arguments to defend their positions. DOR (HK) (talk) 04:24, 17 August 2009 (UTC)[reply]

olympic cities

What is the least populous (at the time) city that has hosted the modern (1896 AD an later) Summer Olympic games? Googlemeister (talk) 19:49, 14 August 2009 (UTC)[reply]

Athens only had 123,000 in 1896. The next smallest seems to be Helsinki, than Saint Louis75.41.110.200 (talk) 20:56, 14 August 2009 (UTC)[reply]
Antwerp would have had a rather small population when it held the Olympics. It may have not been much larger than St. Louis or Helsinki at the time. --Jayron32 04:47, 15 August 2009 (UTC)[reply]
According to our own article it had over 300 000 in 1925, so it was probably similar when it held the Olympics. Adam Bishop (talk) 02:25, 16 August 2009 (UTC)[reply]

Robert Mugabe suspect of killing people?

Is it just rumours or Robert Mugabe have actually kill millions of peoples? Everyone and teachers have said mugabe have kill millions of peoples. Have other national leaders done stuff like this? Is it true or they just hear wrong stuff?--69.229.39.33 (talk) 20:51, 14 August 2009 (UTC)[reply]

Mugabe is claimed to have directed policies which led to many starving (and to have had opposition members disappeared), not to have personally killed them. Many other leaders have also done such thing unfortunately. 75.41.110.200 (talk) 21:19, 14 August 2009 (UTC)[reply]
I don't know of any accusations of his personally killing anyone, but there are plenty of accusations of him ordering killings. The veterans from the revolutionary war tend to go around beating people to death to make sure they vote the right way, that kind of thing. (Obviously, none of these accusations has been proven in a court of law, but since Mugabe appoints the judges that doesn't mean a great deal.) --Tango (talk) 00:43, 15 August 2009 (UTC)[reply]

August 15

Islamization in Ancient Mongolia

Why the arabs and other muslims didn't islamize ancient Mongolia, but islamize Central Asia and islamize some people in China? 174.114.236.41 (talk) 01:33, 15 August 2009 (UTC)[reply]

This is an interesting question and one for which I don't have an authoritative answer. However, if you take a look at our articles Islam in Mongolia and Buddhism in Mongolia, you will find pieces of the answer. I would suggest that Islam has always been primarily an urban religion, and that until recent times, the Mongols were a people without cities. (China and Central Asia, by contrast, have a strong urban tradition dating back to pre-Islamic times.) Certainly, nomadic Arabs and Berbers embraced Islam, but the centers of Islamic learning and religious culture were always cities: first Medina and Mecca, then Damascus, Baghdad, Cairo, and others across the Islamic world. Buddhism took root in a number of regions with little urbanization, such as Tibet, in part because of its monastic culture, which created centers of religious learning and culture in dispersed rural locations. I think that it is no accident that Mongolia adopted the very monastic Tibetan version of Buddhism. I suspect that Islam might have caught on among the Mongols if the Mongols had remained in control of the cities of northern China, which could have become centers of a Mongolian Muslim culture. However, they were driven out of those cities in the 14th century and returned to nomadism and shamanism. Mongolian tribal leaders later introduced Tibetan Buddhism, which they used to legitimize their rule. They set up monasteries around Mongolia, and Buddhism became an integral part of Mongolian culture. Marco polo (talk) 02:18, 15 August 2009 (UTC)[reply]
There was some Islamization in the general greater Mongolia area, but this was followed by spread of Buddhism. According to The Rise of the WestbyWilliam H. McNeill:
"These Russian successes in the western steppe and in the Siberian forests, together with the progress of Lamaism in the central and eastern steppe, combined to roll back the northern frontier of Islam very substantially in the sixteenth–seventeenth centuries. The great Moslem powers made no serious efforts to check this trend of events. Persia, the natural guardian of the gate upon the central steppe, was heavily engaged against the Ottoman empire and had no energy to spare; while the Uzbek empire of Bokhara, by espousing a rigid and narrow Sunni orthodoxy, effectively repressed the Sufi missonary spirit which in previous centuries had won for Islam its main successes among the steppe peoples. ... The cultural and religious vacuum thus created was duly filled by Tibetan Lamaism."
-- AnonMoos (talk) 02:40, 15 August 2009 (UTC)[reply]
Is it possible that they just weren't very interested in monotheism? I mean, I know some of them were Christians and (at least Nestorian) Christianity was generally tolerated, but there is that story that when the pope tried to proselytize to them, Kublai (or whoever it was) said he didn't want to worship their god, and instead the pope should worship him. Maybe something similar happened with Islam. Adam Bishop (talk) 12:06, 15 August 2009 (UTC)[reply]

Church use of popular music

A copyrighted piece of music cannot be freely used for commercial purposes, but what about non-commercial purposes? Would it be legal for a church, for instance, to set hymns to popular copyrighted tunes of the day? My feeling is that this would be illegal, but I'm not positive. I am not requesting legal advice. I am merely curious. LANTZYTALK 02:30, 15 August 2009 (UTC)[reply]

Lincoln Brewster wrote alternate lyrics to Leonard Cohen's song Hallelujah; (his version is called Another Hallelujah). The music I have seen for it always gives joint credit to the two. Presumably, Brewster pays royalties to Cohen and obtained legal permission to use his tune first. In general, you are always required to get permission to record OR publicly perform music whose copyright is owned by someone else; this includes music in alternate settings, cover versions, alternate lyrics, song samples, etc. etc. See Performing rights for more info.
In order to ease the use of copyright christian music in church services, most Contemporary Christian music is liscenced by Christian Copyright Licensing International to allow non-profit performance in worship services. It is somewhat akin to the Copyleft liscences used by Wikipedia such as GFDL and CC-BY-SA, in that it allows material to be used in a limited setting as long as proper credit is given. Churches and congregations pay a fee to have access to CCLI liscenced music for use in worship services. CCLI then pays a portion of this money to the copyright holders. However, this liscence must be specifically applied by the copyright holders, and does not generally apply to music to which it is not specifically applied, so it is usually limited to Contemporary Chirstian music; popular music which is not covered by CCLI would be ineligible. --Jayron32 04:43, 15 August 2009 (UTC)[reply]
I don't think the comparison to copyleft licenses is particularly apt. If I understand your explanaition and the article correctly, the CCLI is a licensing body and you do have to pay a fee (small that it may be) and they do pay royalties, even for non commercial religious use to get a license (without paying you have nothing). If they did allow non commercial only use without a fee, a better comparison BTW would be CC-BY-NC-SA Nil Einne (talk) 09:12, 15 August 2009 (UTC)[reply]
Genuinely speaking non-commercial usage doesn't make a great difference as to legality of using copyrighted material without the owners permission. As mentioned in a number of places in fair use in relation to US law but I think this also applies to many other jurisdictions, while whether or not something is used commercially can make a difference, often it does not. More likely the big effect would be on the sanctions. (Of course a copyright owner is also often less likely to take action if it's non commercial, but that doesn't mean your legally allowed to do it.) Some people may specifically allow their material to be used non commercially, but that's a copyright owner granting permission rather then something explicitly allowed in law. Nil Einne (talk) 09:12, 15 August 2009 (UTC)[reply]
You could ask these people. Dismas|(talk) 17:40, 15 August 2009 (UTC)[reply]
Most definitely they must get permission — for example, my church's psalter includes copyright notices on all copyrighted tunes. One example is a recently-reharmonised version of the mediæval tune "This Entris Nyght" (click here to hear a slightly different version of the tune), which includes a comment: "By permission of Oxford University Press". Nyttend (talk) 01:27, 16 August 2009 (UTC)[reply]
I should note — the first time I saw a tune used through CCLI (in a college chapel program), I wondered if it were a Creative Commons license; a pity that the acronyms are the same. Nyttend (talk) 01:28, 16 August 2009 (UTC)[reply]
Nil Einne is right about the CCLI - and there are equivalents in other countries. You should check for your situation - things vary by country. In the UK I believe there are exemptions for works used solely as part of a 'service of worship', but I can't give you the details. If you are associated with your church there is probably someone who has done research into it - or failing that someone in your denomination's central office. DJ Clayworth (talk) 13:47, 17 August 2009 (UTC)[reply]
I'm eagerly awaiting a Hymn written to the tune of "Why Don't We Do It in the Road?" Baseball Bugs What's up, Doc? carrots 13:59, 17 August 2009 (UTC)[reply]
Yeah an example I've just read about (came across it for unrelated reasons) is [3] which includes some details on the fees. This isn't for Christian music but it does illustrate that this sort of licensing is actually the way this thing usually works. (It makes a lot of sense, a bar, gym, supermarket etc doesn't of course need to negotiate with each artist to be able to play songs.) On the subject of exemptions our article on the CCLI says "It is important to note that the performance of works in copyright (for example, playing music) as part of an act of worship is specifically exempted from copyright laws in several countries[5][6]" and links to references on Canadian and US law. In the US case at least, the exemption appears to cover the performance, but may not cover things like printing booklets with the lyrics, displaying them on screen etc which is part of the stuff the CCLI does Nil Einne (talk) 15:20, 18 August 2009 (UTC)[reply]

Abdoulaye Wade's famous shaven head

Since when have Abdoulaye Wade became hairless. have he once have a hair, or when his hair starts to gray or white he shaves it completely, so he won't look his age or he was bald when he was in his 20s?--69.229.39.33 (talk) 03:21, 15 August 2009 (UTC)[reply]

Well, he had hair at the back of his head in 2007. See this site for pictures from an EU conference in Portugal. (Click on the photo at the top left to enlarge it.) // BL \\ (talk) 03:42, 15 August 2009 (UTC)[reply]
Wow he had full black hair in 2007 in Europe, but a year ago at Hu Jintao's FOCAC he is completely egg baldy.[4] [5]--69.229.39.33 (talk) 17:23, 17 August 2009 (UTC)[reply]

What is the difference between empiricism and mechanistic materialism?

What is the difference between empiricism and mechanistic materialism? The both seem similar derived from Francis Bacon and Newtonian mechanics. I suppose French materialism was more atheist. Who were some mechanistic materialists? --Gary123 (talk) 04:31, 15 August 2009 (UTC)[reply]

Too sleepy to say anything about the first question, but the first names which leapt to my mind were Baron d'Holbach and Julien Offray de La Mettrie - and see the others in French materialism.John Z (talk) 10:55, 15 August 2009 (UTC)[reply]
A difference of the focus. Empiricism states that our knowledge is conveyed through the senses, sensual experience. Materialism supposes that all is based on the existence of a material world and that there is nothing beyond. One might say that empiricism presupposes materialism - if we suppose that sensual experience comes with material organs such as eyes - subject mater that does something with waves of light and nerves. If you want to be an empiricist without the materialist bias you have to chose Empiriocriticism as proposed by Ernst Mach around 1900. Mach assumes that we do indeed create our knowledge on the basis of sensations, experiences we have to interpret. How we get these sensations and what creates them, Mach says, is a matter of models we have to create. Empiricists, so Mach, work on the model that they get these sensations through such material objects as eyes. (One might just as well dream and have the same sensations). Empiricism and Empiriocriticism are closely related since Mach continues to state that the model solution Empiricists have developed (material world gets in contact with senses, creates experiences), is indeed extremely useful - more useful than any other. If our sensations were just a dream we would still use the same model to interpret them.
Materialist criticism of Empiriocriticism as formulated by Lenin in 1908 states that one has to start with supposing matter. Empiricism is already a step towards individualism with all its theorising about how the individual perceives the world. Materialism is the foundation to explain why we have a collective experience of the same reality. Empiricism, by contrast, is (so Lenin) doomed to lead to Empiriocriticism and Solipsism, the theory that there is a single conscience imagining it all. Natural scientists tend to work with Mach's Empiriocriticism, whenever they assume that they create models, theories rather than the truth - models that stand certain tests. The idea is useful as it allows a pluralism of models to evaluate different fields of data. --Olaf Simons (talk) 11:59, 15 August 2009 (UTC)[reply]
A simple way to think about it might be as such: Empiricism is an epistemological philosophy—an answer to the question of how do you know what is true about the world? Mechanistic materialism is an ontological standpoint—a statement about how the world is. The answers they give are somewhat similar but not necessarily—you can be an empiricist (and believe knowledge can be made tangible only through experiment) without being a materialist (believing all of nature is but matter and motion). You could, hypothetically, also be a materialist who is not an empiricist—that is, you believe all of the world is but matter and motion, but feel that only abstract philosophical analysis can get to true truth, or something like that, I imagine. --98.217.14.211 (talk) 15:26, 15 August 2009 (UTC)[reply]

Bangladesh student wing

Is Bangladesh the only nation whose political parties have student wings? What about India, Pakistan, Canada and U.S.? —Preceding unsigned comment added by 67.208.18.71 (talk) 13:24, 15 August 2009 (UTC)[reply]

Political parties in the UK have groups in most universities, I think they are linked through some kind of junior party or something. --Tango (talk) 13:30, 15 August 2009 (UTC)[reply]
Political parties have student wings in many countries. Certainly, the system is identical to Bangladesh in say India and Nepal. In Sweden national political parties generally have student wings. --Soman (talk) 13:33, 15 August 2009 (UTC)[reply]
While not specifically for university students, in the United States there are the Young Republicans and the Young Democrats of America. -- 128.104.112.102 (talk) 18:19, 15 August 2009 (UTC)[reply]
And, of course, there are university-level versions: College Democrats of America, College Republicans, and College Libertarians. Joe (talk) 19:53, 15 August 2009 (UTC)[reply]
Category:Youth wings of political parties by country should be of use to you; thence one can access our articles about the individual youth/student wings of parties in Canada, India, and the United States (we haven't a category for groups associated with Pakistani parties, but they do, I know, exist). Joe (talk) 19:53, 15 August 2009 (UTC)[reply]

See List of Indian student organizations and its related category. --Soman (talk) 10:21, 16 August 2009 (UTC)[reply]

And, in China, the Communist Youth League. DOR (HK) (talk) 06:25, 17 August 2009 (UTC)[reply]

Christ killers

Why are Jews most often blamed for being "christ-killers" and yet Italians - are they whose left from the roman empire? - go blame free. I understand that Jews played apart in Jesus' arrest but they never actually hung him up, yet people justify Jews being christ-killers as a reason for anti-Semitism. Any thoughts on why? Thanks, Thanks, Hadseys 19:24, 15 August 2009 (UTC)[reply]

I might be overreacting, but I can see this question becoming a debate.86.5.193.207 (talk) 19:40, 15 August 2009 (UTC)[reply]
Lenny Bruce had a schtick built around this.
"Hey, why did you Jews kill Christ?..."
"Eh, you know how it is, one of those parties got out of hand..."
Rhinoracer (talk) 19:56, 15 August 2009 (UTC)[reply]
The proper response to "You killed Christ!" is "So thank me, already." —Tamfang (talk) 01:27, 18 August 2009 (UTC)[reply]
Because the Roman Empire eventually converted to Christianity, in part. But you can't expect a whole lot of rationality when it comes to hating entire groups of people for alleged grievances perpetuated by a few of them a thousand years ago. --98.217.14.211 (talk) 19:59, 15 August 2009 (UTC)[reply]
The rationalization is that Romans didn't want to crucify Jesus but the Jews forced him to do so. So, the Jews get the blame. In reality, those who call the Jews "Christ-killers" do not actually care about Jesus (who taught forgiveness and love). They only care about finding some reason to justify their irrational fears and hatred. -- kainaw 21:15, 15 August 2009 (UTC)[reply]
See Matthew 27:11-25 [6]. The answer to your question is right in the New Testament. The irony is that Christ himself was a jew, as was Matthew, the man who wrote that passage. Wrad (talk) 21:19, 15 August 2009 (UTC)[reply]
It's actually very simple - some Jews turned out to be fantastic bankers and got very rich, some Christians were jealous of said Jews and came up with any excuse they could think of to persecute them. It has nothing to do with religion. (Ok, I'm grossly over-simplifying the causes of anti-Semitism, but you get the idea.) --Tango (talk) 21:49, 15 August 2009 (UTC)[reply]
You are oversimplifying it by giving it a rational reason. Some Christians met some Jews and said "Hey - they are different!" Then, they hated them. Similarly, some Jews met some Christians and said "Hey - they are different!" Then, they hated them. -- kainaw 21:54, 15 August 2009 (UTC)[reply]
You get simple prejudice from just being different. The kind of persecution seen of Jews in the 20th century was on a different scale altogether, though. There is normally a rational reason behind such large scale persecution since it needs to be centrally organised and politicians rarely do something without a reason. --Tango (talk) 01:00, 16 August 2009 (UTC)[reply]
What the passage from Matthew is describing is that the Jews, who were under Roman rule, were given a choice as to who would be freed from being punished. The crowd was given the choice of either Jesus or Barabbas. The crowd asked for Barabas to be set free and when asked what should become of Jesus, they responded by saying that he should be crucified. That's the boiled down version of what I remember of what I learned in my Catholic grade school. Dismas|(talk) 00:01, 16 August 2009 (UTC)[reply]
Jesus was Jewish, and he thought of himself as a Jewish prophet. His first supporters were Jewish, and did not think of themselves as non Jewish, "Christians". But, during the end of the first century, the small community who believed in the resurrection of Jesus progressively became more and more estranged from its Jewish background -especially as a growing part of it was made up of non Jewish people, thanks to the "mission" of Paul. On the other hand, the Jews were more and more hostile to this small community, which was a potential contestant on the resources that gave the non Jewish people who wanted to convert to Judaism (who were numerous at the time), and no longer respected some of the basic laws of Judaism. The Gospels were written precisely at that time: they are the testimony of this estrangement and of the tensions that were growing at the time between the no more Jewish Christians and the Jews. Gede (talk) 00:13, 16 August 2009 (UTC)[reply]
I second Gede. The early proto-Christian movement contained several competing and diverging factions, most prominently the "for Jews and converts to Judaism only" one led by Jesus's relations and closest followers, and the "for Gentiles too, you don't have to be circumcised and follow the dietary (etc) laws" one led by Saul/Paul. The latter won out and in order to avoid offending potential Roman converts, spin-doctored the accounts of Jesus's trial(s) and crucifixion to shift the blame onto "the Jews", although in fact only a minority Saduccean faction within the Sanhedrin had been opposed to Jesus, and the execution had necessarily been carried out by the occupying Roman authorities for an offence (sedition) against them. 87.81.230.195 (talk) 03:16, 16 August 2009 (UTC)[reply]

Credit card merchant fees

I have a credit card... unfortunately for the bank, I actually pay my balance in full on time. My question is: is there a reliable way to check on credit card fees for merchants? I know it changes depending on the service (e.g., restaurant, car repair, etc.). Basically: what I'm trying to figure out is:

  1. is my bank or whoever they outservice it to making any money off me having a credit card that's paid off every month (no fees are thus applied),
  2. do companies generally make money off these customers who pay off their balances? I'm speaking mostly of VISA, though Mastercard or Discover are big in the US too. I actually work on the personal checking/savings side, and I can tell you that banks do not make money on these accounts unless the customer is overdrawing (it is, of course, contingent on VISA debit card use, per above).
  3. how badly am I screwing over the merchants if I use my card for a small charge? I go and buy an energy drink for $2.40. Is there a minor fee charged per transaction for the merchant? Etc.

68.32.4.156 (talk) 20:41, 15 August 2009 (UTC)[reply]

I have set up many credit card processing sites. There are two fees involved. One is a flat-rate fee per "swipe". You can think of it as a transaction fee. I say "swipe" because they get charged if you debit or credit the card. I've seen this swipe fee as low as 15 cents and as high at $1.50. Then, there is the percentage of all charges. This can be very low (half a percent) or very high (I've seen 6.5%). Sometimes there is a cap for extremely large purchases. So, you can figure that when you purchase something with a credit card, the merchant is paying out about 25 cents plus 1% of the charge. On a $2.40 charge, that will be 27 or 28 cents (depending on rounding rules). The charges for running an ATM card are completely different. Also, Discover and American Express charge an extra percentage on top of the standard fees. So, if you make your purchase with one of those you will be costing the merchant even more. -- kainaw 21:11, 15 August 2009 (UTC)[reply]
Processing fees can vary widely depending on the merchant. Larger retailers have more bargaining power and can negotiate lower fees and the credit cards charge on a sliding scale depending on the volume of business the retailers do. I'd be surprised if Wal-Mart, for example, was paying 1% of each credit card transaction. —D. Monack talk 21:33, 15 August 2009 (UTC)[reply]
I'd be surprised if WalMart was not operating as their own credit card processing company. They are currently moving towards creating their own bank by inviting local banks to set up shop inside WalMart. It won't be long before all those local banks suddenly become WalMart bank. -- kainaw 21:39, 15 August 2009 (UTC)[reply]
However you pay for your energy drink, the merchant is going to end up paying a fee. There are fees for paying in cash for businesses (at least, there are in the UK, I imagine there are in the US - in fact, does the US have free personal banking?). --Tango (talk) 02:00, 16 August 2009 (UTC)[reply]
In the US, banks are charged for holding customers' deposits.DOR (HK) (talk) 06:30, 17 August 2009 (UTC)[reply]
I don't know, but I have seen many small-time merchants strongly urge customers to pay in cash for small purchases (under $5, usually), because the credit fees on such transactions, they say, make it unprofitable. --98.217.14.211 (talk) 15:13, 16 August 2009 (UTC)[reply]
There is no charge unique to cash purchases in the US. Also, free personal checking is very common. Most banks offer it. They make a lot of money off charging poor people for not having money. -- kainaw 19:48, 16 August 2009 (UTC)[reply]

Dhabihah and hunting

Is it possible for a Muslim to hunt in accordance with Islamic law? I would imagine that following Ḏabīḥah would make it quite difficult. Even aside from the practical requirements of using a knife, the animal might be frightened to hear the human voice pronouncing the name of God. Conversely, I'd be quite surprised if a world religion effectively prohibited hunting yet permitted the slaughtering of animals. Nyttend (talk) 23:41, 15 August 2009 (UTC)[reply]

What if you only captured the animals (alive) in the field? It seems that that would open up a lot of options. --98.217.14.211 (talk) 00:54, 16 August 2009 (UTC)[reply]
These [7] [8] [9] all suggest hunting wild animals is fine, although you may have to pronounce the name of God before throwing the spear (or whatever) and also have to slit the throat as soon as you can. The later two I think are just a translation of the Islamic Laws, not sure about the first. This obviously doesn't tell us what the Quran says (my guess is you could come up with enough to support many different positions) nor what earlier Islamic scholars opinions on this were. Nil Einne (talk) 16:45, 16 August 2009 (UTC)[reply]
I don't have the book to hand, but I believe it was in the 1937 memoir of Kenya, Out of AfricabyIsak Dinesen, that I read of a safari in which the Muslim servants would not eat the game. Dinensen discussed this with an iman, and he issued a fatwa that the meat shot would be halal if she uttered a prayer to Allah when she pulled the trigger. Her employees accepted this ruling. BrainyBabe (talk) 18:09, 16 August 2009 (UTC)[reply]

August 16

What's REALLY the oldest profession?

Colloquially, whoring is considered to be the "oldest profession", but how true is this? Besides sex--violence, eating, and communication are pretty essential human activities as well. Might the mercenary/hitman, the hunter/fisherman or the bard/poet/entertainer be equally ancient occupations? How do we know that the idea of trading/bartering for sex is so ancient relative to paying for other essential services? Anthropologists, please weigh in.--The Fat Man Who Never Came Back (talk) 01:34, 16 August 2009 (UTC)[reply]

It may depend on what you consider to be a profession. How about leadership? A tribal chief, or whatever, performs a task for other people (leading them) and, presumably, receives some kind of compensation (a share of the food without having had to grow/hunt/gather it himself (or herself), perhaps). Other primates have hierarchies, so I expect humans have for as long as we have been human (which is a subject for debate in itself). --Tango (talk) 01:57, 16 August 2009 (UTC)[reply]
Mother and Father. Wrad (talk) 04:32, 16 August 2009 (UTC)[reply]
I don't think that's a "profession." --98.217.14.211 (talk) 04:42, 16 August 2009 (UTC)[reply]
Landlord?Clarityfiend (talk) 05:58, 16 August 2009 (UTC)[reply]
Flint-knapper.--Wetman (talk) 06:04, 16 August 2009 (UTC)[reply]
It is just a saying but a good case could probably be made that bartering for sex is the earliest form of trade in the animal kingdom. Dmcq (talk) 07:47, 16 August 2009 (UTC)[reply]
It depends what you mean 'trade' I guess. Couldn't the endosymbiotic theory of origins of mitochondrion be considered be considered a form of trade between the eukaryotic (orwhatever) cells and the prokaryotic cells that formed the mitochondria? While this obviously preceeded animal evolution, it was a trade between two cells all animals are dependent on. Many bacteria transfer plasmids although you could perhaps argue this isn't a trade since the donor bacteria isn't guaranteed something in return* and I don't know if this definitely predated animal evolution. *I presume it's likely this evolved as there's a selective advantage to the plasmid being transfered. Of course this is sometimes called bacteria sex anyway... Nil Einne (talk) 16:22, 16 August 2009 (UTC)[reply]

The answer really depends on what is meant by "profession." If it's used in the broad sense of an occupation or career, then the answer is probably flint-knapper (or, if even broader senses are intended, hunter-gatherer). However, profession more narrowly means an occupation that requires considerable training and specialized study, a definition that excludes prostitution, at least at the entry level. My guess would be midwife. John M Baker (talk) 21:35, 16 August 2009 (UTC)[reply]

On the basis of that definition, I'd have to go with Wetman's flint-knapper. DOR (HK) (talk) 06:32, 17 August 2009 (UTC)[reply]

The classic joke has it as follows - the answer is lawyers. In the beginning there was chaos - who do you think caused the chaos? --Dweller (talk) 13:27, 17 August 2009 (UTC)[reply]

DOR - Traditionally, something like flint-knapping would be considered a skilled trade, rather than a profession. Among the distinguishing factors are the relative lack of formality in admission to and organization of the trade and the fact that the trade does not involve the use of independent professional judgment. John M Baker (talk) 15:14, 17 August 2009 (UTC)[reply]
As has been mentioned, a key matter is to distinguish a 'profession' from a 'trade.' I'd pick priesthood. B00P (talk) 19:26, 17 August 2009 (UTC)[reply]
Did any prehistoric religions have an organised priesthood? I think religious leaders would learn in the same way other specialists learn - through apprenticeships. So where do you draw the line between professions and trades? --Tango (talk) 22:36, 17 August 2009 (UTC)[reply]
The boundaries between skilled trades and professions are fuzzy and elusive, and some are difficult to justify. For example, electricians are usually considered to be members of a skilled trade, and our article is explicit on this point, but electricians do meet most of the standards for a profession. I guess the justification must be that electricians do not receive as much formal education as most learned professions; they do have formalized training, restrictive standards on admission, and the use of independent professional judgment.
Priests traditionally are professionals, so they would be another candidate for the oldest profession. However, flint-knappers probably could pick it up by observation and practice, with no real standards to show if they were flint-knappers or not, and no independent professional judgment is involved. John M Baker (talk) 14:21, 18 August 2009 (UTC)[reply]

US Social Security numbers

Will US Social Security numbers be reused after a moratorium period if the first person to have that number dies? That is if they are needed... Or has the US gov't not thought that far ahead? How many numbers will be possible given the current number of digits? Dismas|(talk) 02:06, 16 August 2009 (UTC)[reply]

Given the restrictions, there are, AFAICT, 987,921,198 possible viable SSNs (absent the conditions, there are 1 billion [10^9] combinations). 68.248.234.20 (talk) 02:20, 16 August 2009 (UTC)[reply]
I guess we won't be running out of numbers any time soon then... Dismas|(talk) 04:23, 16 August 2009 (UTC)[reply]
No, and if we did... we could just add another digit, and get 9 billion more combinations. --98.217.14.211 (talk) 04:46, 16 August 2009 (UTC)[reply]
And such an action would cause massive problems for virtually every computerized personnel system. Who then was a gentleman? (talk) 19:41, 16 August 2009 (UTC)[reply]
Considering a birth rate of 14.2 per 1,000, a net migration of 3.05 per 1,000, and a current population of 305 million, roughly 5.2 million new SS numbers are needed each year. This figure doesn't include SSN's of people who migrate out of the U.S. Considering the current population and that the Social Security system is 73 years old, it is likely that nearly half of the available numbers have already been used. By my back of the envelope estimation, all of the remaining numbers will be used within 100 years. —D. Monack talk 03:31, 17 August 2009 (UTC)[reply]
Then we'll have a Y2.1K problem, and thus a whole bunch more employment for computer programmers! Except this time around, they'll be offshore. Baseball Bugs What's up, Doc? carrots 07:31, 17 August 2009 (UTC)[reply]

Are poor people in USA really buying homes?

I have seen several claims that (depending on the time and source) approximately 40-46 percent of poor households in USA own their home. My two dozen co-workers and I each earn within a few cents of minimum wage; none owns a home or has any real hope of owning one. Our ages are well-distributed (twenties through fifties) and none has any children to support (a few of us have adult children, but no minor children to support).

So some of us are wondering how millions in USA who are poorer than we (since none has any children to support, none are considered poor) own homes while we have no hope of doing so.

How do these poor homeowners do it? —Preceding unsigned comment added by 67.42.2.70 (talk) 02:14, 16 August 2009 (UTC)[reply]

Well, that was one reason for the subprime mortgage crisis. People who could not afford mortgages got them anyway. Adam Bishop (talk) 02:22, 16 August 2009 (UTC)[reply]
It was made possible because the US government decided that "everyone should own his own home" and interfered in the housing market to make this happen. Specifically: (1) the government established Fannie Mae and Freddie Mac, government controlled corporations that were charged with buying mortgages from banks so that the banks were able to make additional loans; (2) Fannie Mae and Freddie Mac were (a) exempt from filing requirements so that their activities were partially hidden from the public view, (b) implicitly backed by the US government -- meaning that, if FM and FM were to go bankrupt, it was understood that the US government would bail them out; (3) Congress put political pressure on FM/FM to buy "high risk" mortgages (mortgages to people with spotty credit, low income, and un-documented income); (4) the government put political pressure on private banks to loan to lower income people. The end result was a system in which banks were pressured to, rewarded for, and shielded from the risk of making loans to people who couldn't pay off the loans. Wikiant (talk) 02:26, 16 August 2009 (UTC)[reply]
I can understand that you take some enjoyment in repeating the Republican take on the current crisis as often as possible, but in this case the question wasn't about that at all.--91.148.159.4 (talk) 15:54, 17 August 2009 (UTC)[reply]
(EC with above) There are many things which contribute towards people being able to own their own home. In some metropolitan areas (say Washington DC or LA) it is impossible for all but the upper middle class and richer to own a home; 1500 square foot homes are routinely still selling for half a million dollars or even more; modest-sized homes are still out of reach for most poorer families. However, in other places, it is still possible to buy a similar sized home for less than $100,000. I own a similar sized house in Raleigh, North Carolina with a ~$150,000 mortgage, which puts the monthly payments in range of what rent would be in a similar sized apartment. There are rural areas where similar sized homes could sell for much less, thus there are many places where "poor" people can afford a home that richer people in other areas could not.
Furthermore, home prices have risen faster than incomes over the past 50 years or so; many poor people may have purchased their homes when the value was more within their reach. Take a look at these charts: US median incomes and US median home prices. Just looking at national averages; in 2000 the median income was $50,557 (2007 dollars) and the median home price was $119,600 (2000 dollars), this means that a home was more than 2x a years salary for a person. In 1970, the same income number is $41,620 while the same home number is $65,300 which is only about 1.5x a years salary. While these are very rough numbers, it is simply that in the past, housing occupied a smaller fraction of a person's budget, which means that poorer people could better afford to buy a house. If you bought that house in say, 1990, you also bought it with 1990 dollars, which means that while your income may have gone up due to inflation, your mortgage payments remained constant for that whole time; which is why real estate can be a good investment; your income will rise while your payments remain constant. Thus, a poor person, who we already can see could better afford a house in the past, is in a better position to be still owning that house than a richer person trying to buy a house in TODAYS market. --Jayron32 02:35, 16 August 2009 (UTC)[reply]
I've noticed a few times that when Americans say "average income" they actually mean "average household income" which, with many households being couples, the average individual adult income would be something like half this. 78.144.207.41 (talk) 17:37, 16 August 2009 (UTC)[reply]
My guess is that most of these 'poor' homeowners didn't buy homes while poor. My readings suggest that roughly three-fourths of poor homeowners do not have a mortgage, leading to the inference that these three-fourths either bought their homes decades earlier when (a) they were working and had above-poverty income, and (b) prices were much lower and homes were more affordable than today. I believe there is also a significant number of poor homeowners who acquired their homes through preferential, non-market processes. For example, years ago I knew a single mother on welfare ('the dole') who bought her home in a declining neighborhood from her grandmother at preferential price and terms. Today I know a poor woman who inherited her home, and surely there are more than a few others similarly situated. Also, I believe that the percentage of poor who own homes has actually declined in recent years, which would suggest the seemingly high rate has not been driven by subprime lending, which I think has gone primarily to working class people earning more than a poverty-level income. (It's pretty darn hard to qualiufy for a mortgage without a job or other aubstantial and consistent income.) —Preceding unsigned comment added by 67.42.2.70 (talk) 02:57, 16 August 2009 (UTC)[reply]
"Own your home" is much different from "could now buy a home." A lot of those poor homeowners are elderly poor whose only savings are the home the live in and that they bought or inherited many years ago. -Arch dude (talk) 11:48, 16 August 2009 (UTC)[reply]
Most poor homeowners fall into one of two categories: 1) People who bought the house when working and not poor but who are now old and poor because their only income is meager social security; and 2) People who inherited their homes or bought them at a modest, nominal price from an older family member. There is probably a third and smaller category of poor people who inherited or were given a little land by family members and who then built their own house (perhaps with the help of handy family members) on that land. I'm guessing that poor homeowners are most common in the rural South where land prices have always been relatively low and where incomes are low by national standards. Marco polo (talk) 16:29, 16 August 2009 (UTC)[reply]

Also, do you (the OP) live in a city? I'm sure home ownership rates, even among poor people, are much higher in rural areas than big cities, because it is much cheaper there. I think city-folk tend to be wealthier on average which would compound the effect. TastyCakes (talk) 16:02, 17 August 2009 (UTC)[reply]

What cartoon is this describing?

An article by Peter Denning (available here) describes the following:

It was in 1964, I think, when I first saw the now-famous New Yorker cartoon of a classroom of the future. The picture shows a tape recorder on each student’s desk and a tape player on the teacher’s table. The machines whir quietly -- and no one is in the room. Thus did the cartoonist skewer the stereotype of the classroom as a venue for transmitting information from teacher’s brain to student’s notebook.

Apparently, the New Yorker staff haven't been able to find this cartoon given this information from the article. Can anyone suggest the actual source of this cartoon? Sancho 02:33, 16 August 2009 (UTC)[reply]

It may have been the New York Times rather than the New Yorker. See this archive [10] I found using a google search for the terms cartoon classroom of the future tape recorder 1964. This seems to have been a hot topic in 1964. This may be a dead end, as it requires paying the NYT to actually see the contents of the archives, but it may be what you are looking for. --Jayron32 02:43, 16 August 2009 (UTC)[reply]
The New York Times article has a number of silly taperecorder-themed cartoons in it, but nothing like the one described. --98.217.14.211 (talk) 04:16, 16 August 2009 (UTC)[reply]
There is also a similar scene in the movie Real Genius which may be referencing that cartoon... --Jayron32 02:45, 16 August 2009 (UTC)[reply]
Well, I did some rather quick searching around on Google Images with a variety of probable phrases. (You can get all cartoons if you select the "line art" option under the "Advanced" menu.) I don't see anything resembling it (and it doesn't sound like a New Yorker cartoon to me, anyway). Dare I put forward that it is not as "now-famous" as this author believes? --98.217.14.211 (talk) 04:40, 16 August 2009 (UTC)[reply]

Hideous mermaid image

I posted this on the Science Desk with no success, despite many valiant attempts from contributors. Here's the original query: Ages ago, I found on the internet a black and white illustration, possibly an engraving, of a skrinkled up little corpse of a purported mermaid - not the Feejee Mermaid, but one less human, more ghastly. I wanted to use it on the cover page of something I'm writing, but foolishly I seem to have deleted it from my files. Can anyone find it for me? (I'm thinking now it may not have been a mermaid after all, but just some random scientific curiosity from a wunderkammer somewhere).

Thanks in advance. Adambrowne666 (talk) 02:36, 16 August 2009 (UTC)[reply]

I found this on Flickr

http://www.flickr.com/photos/33338389@N06/3104510406/...

hotclaws 07:20, 18 August 2009 (UTC)[reply]

Why'd they rename it Dominican Republic?

The answer isn't available anywhere on Wikipedia! I want to change that. Why did the revolutionaries rename it La Republica Dominica vs. Santo Domingo or something else? Why? NickDupree (talk) 05:01, 16 August 2009 (UTC)[reply]

Probably for the same reason that other countries are named with descriptive names like "Czech Republic" (republic of the Czech people) or "United Kingdom" (a kingdom formed of 4 substituent countries). The capital city is "Santo Domingo" and the people are called "Dominicans" so the name that best describes the "republic of the people who are called Dominicans" is the "Domincan Republic". It's probably so simple an explanation that it does not bear elaborating on. Many countries official names carry some form of "<blank> Republic" or "Republic of <blank>". For some states, there is a more common form of the name, for example "China" as a synonym for "People's Republic of China", which is the real name. For other states, there is no "common" synonym, so we use the full official name "Czech Republic" or in this case "Dominican Republic". --Jayron32 05:29, 16 August 2009 (UTC)[reply]
Prior to independence, they were Spaniards; DR was the colony of Santo Domingo, then Spanish Haiti, and I was not aware their people were called "Dominicans" then. If so, when did they begin calling them dominicans? And Why? NickDupree (talk) 05:39, 16 August 2009 (UTC)[reply]
According to the article, they actually named it『República Dominicana』(Dominica is a completely separate Caribbean island). That name uses an adjective form corresponding to the noun "Domingo"... AnonMoos (talk) 05:39, 16 August 2009 (UTC)[reply]
Ah, I see. They are Santo Domingo-ans, essentially, as that land was the colony of Santo Domingo. NickDupree (talk) 05:43, 16 August 2009 (UTC)[reply]
I found a book from 1830 written by a Cuban, which in one place refers to the people of Santo Domingo as "los dominicanos" (it doesn't seem to be a reference to the Dominican order). So the usage apparently predates independence. --Cam (talk) 15:56, 16 August 2009 (UTC)[reply]
Just to clarify, the city of Santo Domingo was named after Saint Dominic. The Spanish version of Saint Dominic's name is Santo Domingo. The Spanish name Domingo is derived from the Latin Dominicus, so the Latin version of Santo DomingoisSanctus Dominicus. Spanish often uses the original Latin form as the basis for adjectives related to a Spanish noun. Hence the inhabitants of Santo Domingo were known as dominicanos. Marco polo (talk) 16:23, 16 August 2009 (UTC)[reply]
As does English, often. Not that I was ever consulted (*sniff*), but it was decided back in the early days of the settlement that the people of Melbourne would be called not Melbournians, but Melburnians, from the presumed Latin version of the name. -- JackofOz (talk) 21:08, 16 August 2009 (UTC)[reply]
Proleptic etymology, as it were. —Tamfang (talk) 03:15, 18 August 2009 (UTC)[reply]

What was Hitler's opinion of Republicanism?

I was surprised to read in Hitler's table talk that he criticized Napoleon for making himself emperor, and believed aristocratic republic were the best government? Was the Third Reich considered a republic? Did Hitler see the Reich as a third way between republics and monarchies? --Gary123 (talk) 05:10, 16 August 2009 (UTC)[reply]

The Hitler government was clearly republican. Republican is not synonymous with liberal democracy; pretty much any government that is not a monarchy is a republic. Hitler and the Nazi party were elected to their position; lots of republics have heads-of-state with indeterminate terms of office, so that isn't really the dividing line between republic and monarchy. Even most dictatorships are nominally republics. Its a very inclusive term that can be applied to any government where access to executive power is not restricted to a noble/royal class.--Jayron32 05:21, 16 August 2009 (UTC)[reply]
See the first sentance of our article Republic. "A republic is a form of government in which the head of state is not a monarch[1] and the people (or at least a part of its people)[2] have an impact on its government." This broad definition clealry includes the Third Reich. --Jayron32 05:23, 16 August 2009 (UTC)[reply]
Hitler held a great contempt and suspiction against monarchists and the nobility, even if some high ranking Nazis belonged to the nobility (like von Ribbentrop). This can be seen as one of the differences between Nazism and Fascism (Nazism is sometimes seen as a sort of Fascism, but there are too many differences for this to be true, I think). Fascism in Italy and Spain was monarchist and used the monarchy as a symbol for the nation and a link to the past. Nazism was more radical and wanted to create something new. So, to add this to the previous answers, yes, Nazi Germany was a republic. E.G. (talk) 05:31, 16 August 2009 (UTC)[reply]
Practically speaking, how was Hitler different from a Monarch? Baseball Bugs What's up, Doc? carrots 13:27, 16 August 2009 (UTC)[reply]
Practically speaking he did not even have a son or a daughter, a line of succession... --Olaf Simons (talk) 14:44, 16 August 2009 (UTC)[reply]
In the Monarch article, it doesn't seem to establish that as a requirement. However, maybe the term "Emperor" fits Hitler better anyway. Baseball Bugs What's up, Doc? carrots 14:52, 16 August 2009 (UTC)[reply]
What you are doing is confusing a monarch with an autocrat. Some monarchs are autocrats—some are not (think of the UK royals). Hitler was an autocrat, but he was not a monarch. It was not a hereditary rule, which is usually what is defining to a monarchy (elective monarchies being the exceptions). --98.217.14.211 (talk) 14:57, 16 August 2009 (UTC)[reply]
Monarchy merely means that the executive authority of the government rests in a closed "royal" or "noble" class which no one has access to except those with a hereditary claim to royalty or nobility. Even in elective monarchies such as Poland or the Holy Roman Empire, the monarch had to be chosen from among eligible members of the royal classes; the Kingdom of Poland, for example, often elected its monarchs from younger sons of other ruling houses from across Europe. The deal with monarchies is that the access to power is closed UNLESS you are born into the ruling class. Republic refers to any government where theoretically ANY member of the society could be the Head of State; that is being born into a certain class is not a requirement for access to power. That's why the Third Reich is a republic; though not a democratic republic. Hitler's rise to power was not a function of his class at birth but of his own personal power and ambition and skills, without regard for his class. Democracy is not a requirment for a state to be a republic, merely that the executive power is not closed to anyone who isn't royalty. --Jayron32 00:16, 17 August 2009 (UTC)[reply]
In principle you are right, but Nazi Germany was not a monarchy either and for the reasons named above it was more close to be called a republic than a monarchy, if these two were the only options to sort by. – Also, it was not Fascist, it was Nazist. There is a difference between these two ideologies (which has been discussed on this reference desk before not long ago, see Wikipedia:Reference_desk/Archives/Miscellaneous/2009_July_26#Nazism_vs._Fascism). E.G. (talk) 07:30, 17 August 2009 (UTC)[reply]

If Hitler ever said that about Napoleon, I doubt he meant anything serious with it. If you google "republic" on site hitler.org/writings/Mein_Kampf/, you can see that whenever he uses the word, it is in a negative context. He didn't consider himself a "republican", but an advocate of a "Reich": "That is why we must face the calculators of the materialist Republic with faith in an idealist Reich." "In order to look upon such a deed as abhorrent one must have the republican mentality of that petty canaille who are conscious of their own crime." All in all, when he heard "republic", he thought "Weimar republic", and when he heard "Weimar republic", he thought "Jews, Commies, democrats, traitors" (see especially hitler.org/writings/Mein_Kampf/mkv1ch11.html).--91.148.159.4 (talk) 20:23, 16 August 2009 (UTC)[reply]

This is, incidentally, where the Fascism really comes in. Fascism is primarily an idealist philosophy, and opposes itself to materialist ones (even while it reaps their technological rewards). --98.217.14.211 (talk) 20:48, 16 August 2009 (UTC)[reply]
That is why I was so surprised by Hitler's table talk, where he claimed he supported an aristocratic republic like Venice, with a weak people's assembly, and an appointed senate that elected the Fuhrer. He also claimed to support a strict separation between executive and legislative branches, and military role in politics. While nearly all regimes call themselves republic, I would not have thought hitler would have valued republicanism even as a theory to aspire to. Does anyone have more details on this? I have not read Hitler saying anything similar in any other work. Is it possibly a forgery?

This is the passage from Hitler's Table Talk I'm referring to:

"As regards the government of Germany, I've come to the following conclusions:

1. The Reich must be a republic, having at its head an elected chief who shall be endowed with an absolute authority.

2. An agency representing the people must, nevertheless, exist by way of corrective. Its role is to support the Chief of State, but it must be able to intervene in case of need.

3. The task of choosing the Chief shall be entrusted, not to the people's assembly, but to a Senate. It is, however, important that the powers of the Senate shall be limited. Its composition must not be permanent. Moreover, its members shall be appointed with reference to their occupation and not individuals. These Senators must, by their training, be steeped in the idea that power may in no case be delegated to a weakling, and that the elected Fuehrer must always be the best man.

4. The election of the Chief must not take place in public, but in camera. On the occasion of the election of a pope, the people does not know what is happening behind the scenes. A case is reported in which the cardinals exchanged blows. Since then, the cardinals have been deprived of all contact with the outside world, for the duration of the conclave! This is a principle that is also to be observed for the election of the Fuehrer: all conversation with the electors will be forbidden throughout operations.

5. The Party, the Army and the body of officials must take an oath of allegiance to the new Chief within the three hours following the election.

6. The most rigorous separation between the legislative and executive organs of the State must be the supreme law for the new Chief. Just as, in the Party, the SA and the SS are merely the sword to which is entrusted the carrying-out of the decisions taken by the competent organs, in the same way the executive agents of the State are not to concern themselves with politics. They must confine themselves exclusively to ensuring the application of laws issued by the legislative power, making appeal to the sword, in case of need. Although a State founded on such principles can lay no claim to eternity, it might last for eight to nine centuries. The thousand-year-old organisation of the Church is a proof of this—and yet this entire organisation is founded on nonsense. What I have said should a fortiori be true of an organisation founded on reason."

--Gary123 (talk) 22:30, 16 August 2009 (UTC)[reply]

Well, as I just stated some lines above, Hitler was not a Fascist, he was a Nazi (also see the discussion from this July which I linked to). Even if there are similarities between the two ideologies, there are many differences too. E.G. (talk) 07:38, 17 August 2009 (UTC)[reply]

Funny. The quote sounds like a lot of nonsense ... which IMO proves its authenticity. The authority of the Chief of State is "absolute", yet this authority is held in check by "an agency representing the people". The individuality of the Senators doesn't matter, yet each individual must be capable of distinguishing "weaklings" from "best men". "The executive agents of the State are not to concern themselves with politics" - so either the Chief does not concern himself with politics (absurd), or he is not part of the executive branch but rather stands above everything (making the separation of powers meaningless), and "executive branch" actually means "executioners" (this seems very likely, given the obsessive talk of swords and the fact that he thinks the Party's "executive branch" are the SS and SA!) - so he is really saying "I don't want the guys with weapons to displace me from power". Finally, the state supposedly has a separation of branches of power and a system of checks and balances, and this is exemplified with the ... Catholic Church! "Table talk" is a very accurate description for this. I doubt that he was capable of seriously contemplating the future of his Reich beyond his own fate. In this respect, history proved him right, of course.--91.148.159.4 (talk) 17:17, 17 August 2009 (UTC)[reply]

List of Libraries

I'm trying to find a list of the 100 (just an indicative number) biggest libraries of the world. Not only English/American libraries. I know Wikipedia has a List of national libraries, but it doesn't list their size. Even searching in their own articles doesn't always give a reliable valuation: how many items are kept in Biblioteca Nazionale Centrale di Firenze? I don't konow! --151.51.38.205 (talk) 15:49, 16 August 2009 (UTC)[reply]

You may be able to find something on the Worldcat site. Worldcat is the unified online "card catalog" for 71,000 libraries. I'm not sure how to ask Worldcat for a list by size. -Arch dude (talk) 15:55, 16 August 2009 (UTC)[reply]
List of the largest libraries in the world on Listphobia. No sources for the numbers though. --Saddhiyama (talk) 20:01, 16 August 2009 (UTC)[reply]
The list I have posted only mentions the number of books in the libraries and excludes documents, prints etc. Now we only need to list the next 90. I can contribute by mentioning that the Royal Library of Copenhagen holds 4,7 million books. Though what position it has on the list I do not know. --Saddhiyama (talk) 20:09, 16 August 2009 (UTC)[reply]
This page has some information about the size of several other university and national libraries. --Saddhiyama (talk) 20:13, 16 August 2009 (UTC)[reply]
Hm, the fact that some libraries only mentions the number of items, while others only mentions the number of volumes (books), makes it somewhat difficult to create an accurate list. --Saddhiyama (talk) 20:25, 16 August 2009 (UTC)[reply]

Using debit or credit card in supermarket - how much info can they get about me?

Would using such a card give them an electronic record of my name, as well as identifying all my visits and purchases at the supermarket by keeping a record of my account number, for example? I'm just curious about this regarding data warehousing etc. 78.144.207.41 (talk) 17:31, 16 August 2009 (UTC)[reply]

It likely depends on the location. In some locations that will probably be a privacy violation unless you've signed something agreeing to it which you obviously haven't if it's a random store. (For a member card, things will obviously be rather different) Nil Einne (talk) 17:38, 16 August 2009 (UTC)[reply]

Agricultural Subsidies

Where can I find out how much the US Government has spent on agricultural subsidies? --Elatanatari (talk) 17:49, 16 August 2009 (UTC)[reply]

I typed "how much does the U.S. government spend on agricultural subsidies" into google and it gave me this: [11]. There's several useful links there. Google is a wonderful thing. --Jayron32 00:07, 17 August 2009 (UTC)[reply]

Yeah, I've used it ALOT. But I'm trying to figure out how much in total over the last 66 years the US government has spent on Agricultural subsidies. That's what Google hasn't given me.--Elatanatari (talk) 01:22, 17 August 2009 (UTC)[reply]

You will find some tables at this site, including outlays for agriculture. Of course, total outlays may be greater than total subsidies per year, depending on how you define the term subsidy. If you need to get more fine-grained than that, you may need to spend some time at a library with a government document collection inputting detailed budget numbers from the pre-digital years. On the other hand, if you are able to access a database of articles in scholarly journals, you may find that somebody has already done this research. Marco polo (talk) 13:11, 17 August 2009 (UTC)[reply]

UK's action in Turks and Caicos Islands

Are there any precedents (recent) for this kind of action - suspending the government for two years, etc. 75.41.110.200 (talk) 18:33, 16 August 2009 (UTC)[reply]

Yes, Northern Ireland (see Northern Ireland Assembly#The modern Assembly and suspensions). Nanonic (talk) 23:03, 16 August 2009 (UTC)[reply]
Something similar happened during the Pitcairn sexual assault trial of 2004, didn't it? Adam Bishop (talk) 00:26, 17 August 2009 (UTC)[reply]

Is there really a TV news station?

I was watching a video on YouTube. The video was about the boiler explosion aboard the SS Norway. One news station in Miami, Florida obtained a home video of black smoke coming out of one of the smokestacks aboard the vessel. The station identified iteself as Just One Station. Is there really a Just One Station? If so, does it have a website or any other information?69.203.157.50 (talk) 22:30, 16 August 2009 (UTC)[reply]

It appears to be a tagline used by stations of the Sunbeam Television family, which includes two stations in Boston and one in Miami. The implication seems to be "This is a story you can see on Just One Station" (i.e. an exclusive news story). See this google search where the line shows up on news carried either by WHDH-TV in Boston or WSVN in Miami. --Jayron32 00:05, 17 August 2009 (UTC)[reply]

August 17

List of Arts and Sciences

Is there any list in English Wikipedia which covers the existing arts and sciences in the world. I have checked lot of lists but couldn't find the required one. I want a complete list, like a partial list shown in Urdu Wikipedia (link bellow)

http://ur.wikipedia.org/wiki/%D9%81%DB%81%D8%B1%D8%B3%D8%AA_%D8%B9%D9%84%D9%88%D9%85

Regards, —Preceding unsigned comment added by 58.27.205.224 (talk) 08:08, 17 August 2009 (UTC) I require a list of arts and sciences like this Acarology Actinobiology Actinology Aerobiology Aerology Aetiology —Preceding unsigned comment added by 58.27.204.62 (talk) 05:38, 18 August 2009 (UTC)[reply]

Indonesian ballot paper of 1955

Is there anywhere online were one can find image (scan or photo) of ballot papers used in 1955 Indonesian elections? --Soman (talk) 08:11, 17 August 2009 (UTC)[reply]

meaning of ww2 japanese submarine designations

hello. what is the meaning (if any) behind the japanese designations for their ww2 subs? like I-xxx, RO-xxx or HA-xxx. german U-xxx, U came from Unterseeboot, american SS-xxx was Submersible Ship, but hard as i searched, i can't find any explanation for the japanese codes.

thank you. —Preceding unsigned comment added by 195.207.101.112 (talk) 08:56, 17 August 2009 (UTC)[reply]

"RO" is『呂』in Japanese. "I" is "伊". "HA" is "波". Together these spell 伊呂波-- in other words, it is the beginning of the iroha poem, akin to calling the lines "A" "B" and "C" in English. Shii (tock) 19:34, 17 August 2009 (UTC)[reply]

Looking for a Captain Cook work

Does anyone have The Journals of Captain James Cook on his voyages of discovery, Volume 3? I am researching Myrmecia esuriens (a species of bulldog ant), and have found an interesting snippet regarding this ant in a google books search. Would someone be able to provide a full quote from the work? Maedin\talk 11:05, 17 August 2009 (UTC)[reply]

One shouldn't get one's sources at second hand, I know; but if you just want to see what the journal says, I think the second quoted excerpt in the "Historical footnote" box on page 7 of this is the passage you're interested in. Deor (talk) 12:45, 17 August 2009 (UTC)[reply]
That's actually extremely useful, thank you very much! Not ideal, but it will have to do for my purposes. Maedin\talk 15:07, 17 August 2009 (UTC)[reply]

Tracing the movements of George VI of the United Kingdom

Hi. I'm developing the Carrow Road article (feel free to help out). I have RS to say that King George VI visited a match at the ground on 29 Oct 1938. It makes sense to me that this took place during a royal stay at (relatively) nearby Sandringham House, but I'd love to be able to verify this. Is there a reliable source (preferably online!) that could help confirm this? The Times royal engagements column? --Dweller (talk) 12:52, 17 August 2009 (UTC)[reply]

Norwich City Council say he was there on that day to officially open the newly built City Hall and then wandered down for the Millwall match. Nanonic (talk) 13:04, 17 August 2009 (UTC)[reply]
That's a nice detail, but I'm still wondering where he kipped that night/the night before. --Dweller (talk) 13:06, 17 August 2009 (UTC)[reply]
Aha! From The Times Oct 15, 1938; pg. 9; Issue 48126; Start column: B ArticleID: CS151597903. "The Royal Visit To Norwich"
"When the King and Queen go to Norwich on Saturday, October 29, they will drive there from Sandringham with Sir Samuel Hoare as Minister in attendance. After laying a wreath on the war memorial, and inspecting ex-Service men and youth organizations, the King and Queen will take up their positions on a dais outside the City Hall. The Lord Mayor will offer to the King the Sword of State, and will then read a welcome to which the King will reply. The King will then declare open the new City Hall, and after the ceremony the Lord Mayor will offer the King a baton to commemorate the occasion.
The King and Queen will take luncheon with the Lord Mayor in St. Andrew Hall, and afterwards the Queen will visit the Norwhich and Norfolk Hospital to open the Geoffrey Colman Memorial. The King will drive alone to Carrow Road football ground to watch the match between Norwich City and Millwall. Later the King and Queen will visit the Norwich Lads' Club and then the Mutual Service Club before returning to Sandringham by car."
and - The Times Oct 31, 1938; pg. 11; Issue 48139; column: A ArticleID: CS184759135. "Royal Visit To Norwich New City Hall Opened, The King's Tribute FROM OUR SPECIAL CORRESPONDENT. "
"The King went to see his first Football League match - that between Norwich City and Millwall - and was given a warm welcome by the large crowd. The ground was beflagged, and the arrival of the King was announced by a fanfare of trumpets. The crowd joined in singing the National Anthem, and the King walked out on to the field of play to shake hands with the players and referee. Then someone started singing "For he's a jolly good fellow," and the entire crowd took up the song. The King remained to watch the play for a quarter of an hour."
HTH Nanonic (talk) 13:27, 17 August 2009 (UTC)[reply]

Now that is really quite outstanding. Thank you very much. --Dweller (talk) 13:35, 17 August 2009 (UTC)[reply]

Terms of the lease of the New Territories, Hong Kong

This may be a difficult one. I have had this question in mind for years, but never found the answer, so I am coming here to see if it is true that the ref desk is doing wonders :-) Here it is: the Convention for the Extension of Hong Kong Territory "was a lease signed between Qing Dynasty China and the United Kingdom in 1898". Did the UK have to pay any sort of annual "rent" for the "lease", or was it just "free of charge"? And if that's too easy, what about Guangzhouwan? Thanks a lot! olivier (talk) 17:14, 17 August 2009 (UTC)[reply]

This thread on Google Answers is all over the place but seems to say there were no payments or rent due. There seem to be numerous references, though two of the references I clicked were bad links. Tempshill (talk) 20:14, 17 August 2009 (UTC)[reply]
The Google Answers thread cites printed sources that may not be available online. The answer offered there, that there were no rent payments, is confirmed by the actual text of the agreement, on pp. 295 and 296 of this source, which does not mention any payment. Now, if you scroll back to page 293 of the same source, you will find the lease agreement for Guangzhouwan, which likewise fails to mention any payment by France. It seems likely that there was none. Marco polo (talk) 20:24, 17 August 2009 (UTC)[reply]
Thank you VERY much! I am impressed by your ability to find these kind of resources. You guys are really doing a fantastic work here! olivier (talk) 09:45, 18 August 2009 (UTC)[reply]

White vs. APL family

Is japanese husband vs. white wife majority of interracial Asian family or is it the other way around. Becasue in my neighbor, husband is Japanese and wife is white. Same thing I saw on TV a Japanese man vs. white women on a date. I thought in general is an white husband vs. APL wife is most common IR family this is what happens in Chinese school TZU Chi orange county by looking at students last name of "olson" "myers" "Valverde" "Rother" obviously tells, and with facial looks I see alot of white dad, and most moms is chinese. What about Philipino or Cambodian faily is IR family work my white dad vs. APL mom or is it APL dad vs. white mom?--69.229.39.33 (talk) 17:39, 17 August 2009 (UTC)[reply]

From my personal observation, it appears the other way around in my part of the US. Googlemeister (talk) 17:48, 17 August 2009 (UTC)[reply]
Wasn't a very similar question asked a few weeks ago? The way the question is phrased and the fact that the guy seems to claim he can tell whether a half Asian has a white mother or father based on looks make me think this he's trolling. But yes, on the off chance that he's not, I would agree that the other way around seems much more common in North America. TastyCakes (talk) 17:51, 17 August 2009 (UTC)[reply]
I think that that portion of the OP was just saying, "I can see from facial features that the child is mixed Asian and white, and I infer from the non-Asian surname that the father was the white parent (assuming parents were married and the child took the father's surname)". WP:AGF. --Sean 13:26, 18 August 2009 (UTC)[reply]
Here is the data that you want.--droptone (talk) 18:32, 17 August 2009 (UTC)[reply]
This is most straight-up link--69.229.39.33 (talk) 19:10, 17 August 2009 (UTC)[reply]

islamic monks?

Does Islam have anything similar to monasteries and monks from Christianity or Buddhism? Googlemeister (talk) 17:52, 17 August 2009 (UTC)[reply]

Whirling Dervishes? --TammyMoet (talk) 17:55, 17 August 2009 (UTC)[reply]
You may be interested in reading Monasticism#Islam and monasticism. Nanonic (talk) 18:10, 17 August 2009 (UTC)[reply]

Elections of Kenya and mozambique

Do Kenya and Mozabique allow leader to run as long as they want or now they set bar of two-terms only election. This one said Mwai Kibaki will be gone by 2012, and Joaquim Chissano last until 2004, and he's only 65 years at then, and he's younger than John Kufuor. is it because of the constitution or those guys want to quit. Yes I know alot of countries only allow two term election, but once they allow them to stay for >20 years. Daniel Arap Moi was in office for 24 years, joaquim Chissano was in for 18 years.--69.229.39.33 (talk) 18:07, 17 August 2009 (UTC)[reply]

Yes Kenya has two-term limit since 1991 (See here [12], I couldn't find that fact on Wikipedia). 75.41.110.200 (talk) 20:26, 17 August 2009 (UTC)[reply]
Mozambique also appears to have a two five-year terms limit. 75.41.110.200 (talk) 20:43, 17 August 2009 (UTC)[reply]

Does Congress (in the US) have a source control system for the actual words going into a bill?

If I can lock a project so nobody can make any changes without my knowing it, is the government doing any better to keep just anybody on the inside from slipping a small sentence into a 1,000+ page document, and if there is something strange in there, to be able to see who added it and when? 20.137.18.50 (talk) 18:25, 17 August 2009 (UTC)[reply]

Do you think you could explain your question a little bit more? I'm a little confused as to what you're talking about. Do you mean can the Government check-in on or control Wikipedia? What do you mean by the word "project?" ~ Amory (usertalkcontribs) 19:11, 17 August 2009 (UTC)[reply]
I don't think the question has anything to do with Wikipedia, but deals with control of the contents of legislation. There have been famous incidents in which either errors or substantive content have slipped into revised versions of lengthy bills as they moved through the amendment and conference processes. In general, "the bill is being pushed forward too fast for anyone to read it" has been a complaint I've heard about various pieces of legislation going back at least to the 1970s and I'm sure earlier. In answer to the original question, I'm not aware of any new methods recently introduced to address the issue. Newyorkbrad (talk) 19:17, 17 August 2009 (UTC)[reply]
Bills are often impossible to read anyway, since they usually consist of diffs like "From section Amendments, Item 1, Sentence 1, strike the words 'shall make no', and insert 'oughtta make a'". I doubt legislators read any bill before passage; pity the staffers. --Sean 13:38, 18 August 2009 (UTC)[reply]
I've not heard of them using anything like a real source control system; just committees and clerks. It can be quite difficult to figure out where a given sentence in a bill came from, and to maintain total control over the text of very long bills. --98.217.14.211 (talk) 19:21, 17 August 2009 (UTC)[reply]
Sounds like a great idea for the benefit of the poor staffers who are tasked with reading them. (I'm not asserting they actually do.) Tempshill (talk) 20:03, 17 August 2009 (UTC)[reply]
Sometimes things will get slipped in, like when Senator Leahy slipped something into a bill to try have Lake Champlaign declared to be the sixth Great Lake. Baseball Bugs What's up, Doc? carrots 20:09, 17 August 2009 (UTC)[reply]
Or, more sinisterly, when Sen. Chuck Grassley reinserted a clause into the Tax Increase and Reconciliation Act of 2005 that had been rejected by both houses. In doing so, he violated not only the spirit of conference committees, but also the entire principle of representative democracy. Sadly, he has yet to be prosecuted of this heinous act. DOR (HK) (talk) 03:20, 18 August 2009 (UTC)[reply]
I suppose so. Did he break any laws in so doing? Baseball Bugs What's up, Doc? carrots 03:51, 18 August 2009 (UTC)[reply]
In general, legislators' official actions are above the law due to separation of powers issues. This gets enforced even in cases of outright felonies, so Grassley's sort of legerdemain has basically zero chance of being prosecuted. --Sean 13:32, 18 August 2009 (UTC)[reply]
While I would describe Grassley's actions as unethical, it is sadly not that uncommon and certainly not illegal. Other conference committee members and their staffs are supposed to keep an eye on these kind of shenanigans. These committees are bipartisan for a reason. —D. Monack talk 09:02, 18 August 2009 (UTC)[reply]
So how were these instances like what you shared of Senators Leahy and Grassley traced back to them? An electronic log, a paper trail, or the old fashioned grapevine? 20.137.18.50 (talk) 13:04, 18 August 2009 (UTC)[reply]
Can I just say that I find it utterly bizarre that an individual legislator can "sneak" a clause into a bill. I've never heard of this ever happening in any other vaguely democratic country. DJ Clayworth (talk) 14:15, 18 August 2009 (UTC)[reply]
Do other countries pass 1,000 page bills, or are they generally kept manageable (which would make shenanigan detection simpler? Googlemeister (talk) 14:26, 18 August 2009 (UTC)[reply]
Don't know for sure about the size although I believe they tend to be shorter but definitely in Westminister parliamentary democracies I know of, bills tend to be fairly narrow in the sense they have a specific focus and don't get the sort of strange almost or literally unrelated stuff usually into bills that are very likely to pass that you seem to get a lot in the US [13] as famously described in The Simpsons episode Mr. Spritz Goes to Washington. I've read a few of these before on wikipedia, perhaps there's even an article describing the practice but can't seem to find it. While omnibus bills do exist, even they tend to have a more specific focus where the you can understand the provisions being part of the bill unlike in the US where they sometimes seem to be just a bunch of stuff stuck together in the hope they all pass. (This doesn't of course mean certain clauses may not be controversial.) The other thing is perhaps the greater adherence to the party structure. Crossing the floor tends to be a lot rare and people tend to vote along party lines and most bills come from the government of the day (even if it's a minority government). This tends to mean (IMHO) it's a lot more difficult for a lone MP to sneak something into a bill and you can be sure if they do, they'll get into deep shit. The government may 'sneak' provisions in at the last minute but even that I think is less likely due to the furore it will cause and in any case this isn't a case of them presenting a different bill then what people voted on earlier but specifically amending the bill with the support of parliament [14]. You can't just change the wording and hope no one notices. (You may be able to at the select committee stage but as I outlined, that's probably a lot less likely.) It's worth remembering that while the actions may not be illegal as mention by Sean, this doesn't prevent the party or parliament stopping them if they wish to. In most cases they could sanction the person for such actions (or at least change the rules so they can) in some way. Also, you could pass a bill to remove the provision that was snuck in. BTW, I can't find any references to what you're referring to about Chuck Grassley, was it this? "Grassley was eventually able to attach an amendment to a piece of legislation that went into effect in 2006, which increased taxes on Americans" Nil Einne (talk) 15:49, 18 August 2009 (UTC)[reply]
It's not always obvious when they "slip in" this kind of legislation. There was a famous case that came up during the 1992 presidential campaign where Ross Perot had lobbied/bribed some legislators to insert language in a general tax cut bill to say something along the lines of, "if you took more than $30,000 in losses between 1974 and 1978, while having enormous ears, on investments concerning a brokerage with initials DGF, and have an absurd squeaky drawl, then you get a rebate". The provision was argued on its merits, but only affected a single person in the whole country --- Ross Perot would have gotten a check from the IRS for $15 million. --Sean 16:59, 18 August 2009 (UTC)[reply]

Mexican vs. white interracial marriage

The OP above link was about Asian-white families. I found no source about mexican vs. white interracial marriage. Is is mexican-white families is it most common for white husband vs. Mexian wife or is it most common for mexican husband vs. white wife. This is kinda dubious to me becasue in general males skin seems to be daker than females skin.--69.229.39.33 (talk) 20:35, 17 August 2009 (UTC)[reply]

Aren't most Mexicans "white people"?!? --Jayron32 20:52, 17 August 2009 (UTC) Scratch that. Did the research. Most people from Mexico are Mestizos of mixed European/Native American descent. Now, are you refering to Mestizo-white marriages within Mexico, or mestizo-white marriages worldwide? Because "Mexican" is not a race or ethnicity; its a statement of national origin, and something like 9-17% of Mexicans are themselves characterized as "white", so it is entirely possible to have a white Mexican marry a white citizen of another country. --Jayron32 20:58, 17 August 2009 (UTC)[reply]
In the U.S. a Mexican-American can be white and Hispanic - and that would not, necessarily, be considered multi-racial. U.S. Hispanics are about equally likely by gender to be in a intermarriage (except for recent Central American immigrants where women are slightly more likely.)[15] Color of skin is not considered a particular factor between genders. 75.41.110.200 (talk) 23:11, 17 August 2009 (UTC)[reply]
I mean Mexican Hispanic versus native US intermarriage. Which one os most common. A Hispanic husband vs white US wife or white US husband vs. hispanic wife.--69.229.39.33 (talk) 01:41, 18 August 2009 (UTC)[reply]
Hispanic and white are not mutually exclusive categories. Hispanic is a U.S. census designation which describes a person's linguistic background where as White describes ones ethnic background. It is possible to be Hispanic and White simultaneously (see Isabel Allende, as well as Hispanic and Black (see David Ortiz) or Hispanic and multiracial. Your question is still impossible to answer because you are asking for a comparison of two non-comparible things. Its like asking "Did you eat mashed potatos for dinner or take the train to work". Hispanic and White refer to two different things entirely. --Jayron32 02:04, 18 August 2009 (UTC)[reply]
And I might add that "of Hispanic race" and "native US" are not mutually exclusive either (unless you mean Native Americans). DJ Clayworth (talk) 14:11, 18 August 2009 (UTC)[reply]
Come on, don't be obtuse. He means Mestizo. He means the fact that most people of Mexican origin that one interacts with (at least, in Southern California, where the OP is from) have dark hair and brownish skin. Obviously the lines between the "races" are blurry as always but he's asking about general trends, not biological precision. (Obviously in such a situation you'd get many Puerto Ricans and other non-Mexicans who look similar swept into the same perceptual category, which I'm sure is fine.) I don't know the statistics, and I don't know if they are out there, but I am sure you could do a survey to find out (even if you just used self-identification). Obviously the US Census won't work for this because their categories are not set up to answer questions like this, but lord knows their categories are pretty problematic on a number of fronts. --98.217.14.211 (talk) 15:50, 18 August 2009 (UTC)[reply]
I don't think either are being obtuse. Some earlier questions were asked were Mestizo specifically came up and some explanation of why the OP's question was unanswerable. Despite that, the OP continues to use terminology that is confusing and so it remains unclear precisely what the OP is referring to. Perhaps the OP doesn't even understand the difference, in which case it's wise he or she learns before asking questions since it's unlikely he or she could understand the answers otherwise. If the OP wants to know about Mestizos - white marriages, then he/she should ask, but at the current time hasn't. Nil Einne (talk) 16:13, 18 August 2009 (UTC)[reply]

citing music

I posted this question on the Help Desk (where it belongs) but since it's sort of arcane, I thought I would cross post it here in case any musically-minded editors have some feedback.

In the past, I've worked on the article for the Harvard Glee Club, a choir that has had a lot of music written for it by various composers. I added a list of all such pieces that I could identify. Each of these pieces carries a note that says it was written for the harvard glee club, and I clumsily tried to explain that in the list introduction. In my attempt to get the article "good" status, an editor specifically complained that this list was unsourced. I would like to give another go at getting "good" status but I think this is the only issue from last time that will be hard to fix. Is it really necessary to manually cite each item on the list? What is the difference between doing that and simply stating at the top that each piece carries the dedication note? It's all good faith anyway, since no matter how it's cited, people will have to go look up the pieces to "prove" the citation is correct. The only difference I can identify is that the former will take hours and hours of work. Any feedback would be appreciated! Dmz5*Edits**Talk* 22:59, 17 August 2009 (UTC)[reply]

Judaism Ethnic divisions vs. denomination

Is there a website where they have a chart that shows the ethnic divisions of Judaism versus the denomination of Judaism (e.g. Mizrahi vs. Liberal, Mizrahi vs. Conservative, Mizrahi vs. Reformative, etc.)? —Preceding unsigned comment added by 74.14.118.71 (talk) 23:25, 17 August 2009 (UTC)[reply]

I'm not sure what you're asking for, but these articles might help: Jewish religious movements & Jewish ethnic divisions. Note that denominational differences in Judaism tend to be solely among Ashkenazi Jews. —D. Monack talk 09:08, 18 August 2009 (UTC)[reply]

Districts of West Bengal versus religion

Which districts of West Bengal have significant number of Muslim population? Which districts of West Bengal have significant number of Christian population? Which districts of West Bengal have significant number of Buddhist population? —Preceding unsigned comment added by 74.14.118.71 (talk) 23:33, 17 August 2009 (UTC)[reply]

I do not think the Indian census records its religion data in this way. Shii (tock) 00:09, 18 August 2009 (UTC)[reply]

North Americal Aboriginal Treaties

Is there a compilation of Treaties between the various Aboriginal peoples of the Western Hemisphere (i.e. Canada, the United States, Cnetral America and South America) and the governments of these counties that include Treaties broken by either the Aboriginals or the governments (may as well include Britan, France and Spain in the mix). I am interested in how many treaties have been developed, how many broken and by whom. How does this compare to Treaties made with other Aboriginal peoples (e.g. Australia)? —Preceding unsigned comment added by 209.161.242.32 (talk) 00:39, 18 August 2009 (UTC)[reply]

We have Category:Treaties of indigenous peoples of North America, which is a start, although it is heavily skewed towards the US. Adam Bishop (talk) 01:56, 18 August 2009 (UTC)[reply]
There has never been any sort of treaty between the indigenous peoples of Australia and the rest of the Australian populace. But New Zealand had its Treaty of Waitangi. -- JackofOz (talk) 08:22, 18 August 2009 (UTC)[reply]

Here's a link to the Treaty page of Canada's Department of Indian Affairs: [16]. Treaties are very much alive in contemporary Canadian law, and recent land claim settlements are considered to be modern-day treaties. Treaties benefit from special constitutional protection in Canada. --Xuxl (talk) 14:24, 18 August 2009 (UTC)[reply]

The United Nations also conducted a comprehensive study of treaties with Aboriginal groups, in the 1990s, called the: "Study on treaties, agreements and other constructive arrangements between States and indigenous populations". I don't have the time to go through the UN website, but there should be some documents there covering other countries in the Americas. --Xuxl (talk) 14:27, 18 August 2009 (UTC)[reply]

Caucasian Eyes

If oriental eyes are called Epicanthic Fold. What is the technical word for the caucasian eyes? 174.114.236.41 (talk) 00:42, 18 August 2009 (UTC)[reply]

They are not called an Epicanthic Fold, they have an epicanthic fold. Caucasian eyes do not generally have one. See our article titled epicanthic fold. --Jayron32 01:28, 18 August 2009 (UTC)[reply]

rotten vegetables at protests and theater

Where there vendors that sold the rotten vegetables during protests and theater acts for profit? —Preceding unsigned comment added by 138.163.160.42 (talk) 04:34, 18 August 2009 (UTC)[reply]

In the 19th century, Covent Garden was a theater district and also a flower, fruit, and vegetable market, so I imagine that materials would have been easily to hand... AnonMoos (talk) 09:38, 18 August 2009 (UTC)[reply]

Danish titles

Ok I notice that throughout Danish history the King of Denmark had had many titles. Queen Magrathe II (spell it wrong) renounce all these century old titles except for Denmark's King. I was wondering what were the ALL the titles of she renounced and what were ALL the titles that were claimed by her predeccessor. Forget about the obvious ones, Norway and Sweden. And did Christian I of Denmark's descendant continued to use the title of Count of Oldenburg nominally even though Christian I gave the rule of the County to his younger brother? I read here that Christian IX of Denmark used it. --Queen Elizabeth II's Little Spy (talk) 03:07, 14 August 2009 (UTC)[reply]

Here are the ones I know.--Queen Elizabeth II's Little Spy (talk) 03:07, 14 August 2009 (UTC)[reply]

Anybody?--Queen Elizabeth II's Little Spy (talk) 07:54, 18 August 2009 (UTC)[reply]

Logical fallacy: Good or bad based on who does it?

Hi there. I'm trying to remember the correct name for a logical fallacy that goes like this: "Doing X is only a bad thing if A does it. If B does it, it's a good thing." Does that fall under argument from authority? Regards SoWhy 13:45, 18 August 2009 (UTC)[reply]

There's no fallacy involved. A fallacy is a type of flawed argument, and all you've got there is a pair of statements with no argument to support them. Algebraist 13:50, 18 August 2009 (UTC)[reply]
Whilst Algabraist is right per se, you may mean one of several fallacious arguments. I think you may mean the association fallacy. --Leon (talk) 13:59, 18 August 2009 (UTC)[reply]
There are certainly cases where it is not a fallacy at all. Many people would consider that imprisonment, even killing, is proper when done by the state through a court of law. Hardly anyone considers the same things proper when done by a private individual. DJ Clayworth (talk) 14:17, 18 August 2009 (UTC)[reply]
(To DJ Clayworth) That's true, but I'm not sure if that's what the asker meant. My guess is that they were refering to the association fallacy as that is a common one. (To the asker) If you haven't time to check the link, the association fallacy is that in which anything associated with an individual or group perceived as bad is said to be bad. For instance, driving a car could be said to suggest criminal behaviour because criminals drive cars. This needn't always be a fallacy; the label is used for occasions when it is.--Leon (talk) 14:32, 18 August 2009 (UTC)[reply]
It's no fallacy, it's an assertion of contingency. Getting a heart transplant is good if the patient needs and wants one. It is not so good when they do not. If there were to be any objection to assertions of this nature, it would be more to the tone, perhaps less to the content. Condescension or dictation is rarely welcome. Vranak (talk) 16:04, 18 August 2009 (UTC)[reply]
Would this be an example: two biologists are sitting around discussing the deficiencies of evolutionary theory, but when a fundamentalist Christian joins in the conversation, the biologists recoil with horror and refuse to grant any weight to the objections of the Fundamentalist? If so, then it's a sort of case of an argument from authority, but does not seem like a particularly striking case of it. The problem arises when the insiders require excessive demands on anyone joining their discussion. Chomsky has a section in an article where he discusses how folks in the political world demand degrees to establish your credentials when discussing political matters but mathematicians never objected to Chomsky's papers in their field without the proper set of credentials.--droptone (talk) 16:43, 18 August 2009 (UTC)[reply]

Is there any religion where there are two good Gods?

Is there any religion where there are two good Gods? I know there are certain dualist religions like Manicheanism and Zoroastrianism that have one good and one evil God. And certain polytheistic that have many good Gods. IS there any religion that has only 2 Gods, both of whom are good?--Gary123 (talk) 13:51, 18 August 2009 (UTC)[reply]

Wicca has two, a God and a Goddess. Bettia (bring on the trumpets!) 14:47, 18 August 2009 (UTC)[reply]
At least some Wiccans would disagree with atributing a good (or evil) label to their gods though. Rmhermen (talk) 16:12, 18 August 2009 (UTC)[reply]

Ethnic groups of India vs. Buddhism

So far, I know that ethnic group Bengali has some Buddhist followers, according to Bengali people article. What about ethnic groups Marathi, Telugu, Tamil, Kannada, Malayalam, Assamese, Gujarati, and Oriya? Do they have some Buddhist population? —Preceding unsigned comment added by 76.64.128.24 (talk) 16:27, 18 August 2009 (UTC)[reply]

Bangladeshi Buddhist

So far, Bangladeshi Buddhists celebrate only Buddha Purnima? —Preceding unsigned comment added by 76.64.128.24 (talk) 16:28, 18 August 2009 (UTC)[reply]

Wikipedia sentient?

Is Wikipedia sentient?

Obvious question.. We got sentience when cells got organised into humans, we know that.

How about when atoms got organised into molecules and molecules into cells??

Or when humans got organised into classes, like classes of knowing..

Our neurons got together by talking and made me

and you, and everything

and all that

just lying all around

You want references.. look through your eyes and listen.. —Preceding unsigned comment added by Ayelamb (talkcontribs) 17:11, 18 August 2009 (UTC)[reply]

Oh boy, now we have to add to the disclaimer that Wikipedia's Reference Desk is not a place for original free verse.209.244.187.155 (talk) 17:30, 18 August 2009 (UTC)[reply]

Retrieved from "https://en.wikipedia.org/w/index.php?title=Wikipedia:Reference_desk/Humanities&oldid=308719728"

Categories: 
Wikipedia help forums
Wikipedia resources for researchers
Hidden category: 
Non-talk pages that are automatically signed
 



This page was last edited on 18 August 2009, at 17:30 (UTC).

This version of the page has been revised. Besides normal editing, the reason for revision may have been that this version contains factual inaccuracies, vandalism, or material not compatible with the Creative Commons Attribution-ShareAlike License.



Privacy policy

About Wikipedia

Disclaimers

Contact Wikipedia

Code of Conduct

Developers

Statistics

Cookie statement

Mobile view



Wikimedia Foundation
Powered by MediaWiki